UWORLD 3

Pataasin ang iyong marka sa homework at exams ngayon gamit ang Quizwiz!

This patient has obstructive sleep apnea (OSA), which is characterized by recurrent obstruction of the upper airway during sleep; each nocturnal episode of reduced ventilation causes transient hypercapnia and hypoxemia. These blood gas derangements result in reflexive systemic and pulmonary vasoconstriction, endothelial dysfunction, abnormal venous return and CO, and sympathetic cardiac stimulation. Prolonged, untreated OSA can cause pulmonary HTN and RH failure. Most patients with OSA will develop systemic hypertension d/t chronic sympathetic stimulation and elevated plasma NE levels Patients also lose the normal diurnal variation in blood pressure. Other CVS complications of OSA include -afib (and other arrhythmias) -CAD -sudden cardiac death a. acquired bronchiectasis may be seen in patients with recurrent infection, impaired drainage (eg cystic fibrosis ), airway obstruction (eg, foreign body aspiration), or inadequate host defense (eg, hypogammaglobulinmemia). b. systemic HTN, as seen in OSA, can lead to mild-to moderate left ventricular hypertrophy and impaired systolic and diastolic function. In contrast, hypertrophic cardiomyopathy is an autosomal dominant disease of the cardiac sarcomere characterized by severe myocardial hypertrophy. It is not associated with HTN or SOA c. laryngeal carcinoma is associated with cigarette smoking and heavy alcohol use d. like OSA< narcolepsy can also cause daytime drowsiness. However, narcolepsy is also associated with cataplexy (sudden loss of muscle tone), sleep attacks, sleep paralysis, and hypnagogic hallucinations. OSA is not a risk factor for narcolepsy.

1985 A 54 y.o man comes to the office d/t daytime sleepiness and lack of energy. His symptoms began 6 months ago and he feels "completely drained" by the end of the day. The patient's wife mentions the the snores loudly. BMI is 34. PE shows a narrow oropharynx and large neck ciumfevence. The patient is at increased risk of developing which of the following? a. bronchiectasis b. hypertrophic cardiomyopathy c. laryngeal carcinoma d. narcolepsy e. pulmonary HTN

Contact dermatitis, granulomatous inflammation, the tuberculin skin test and the Candida extract skin reaction are all examples of delayed type hypersensitivity reactions (DTH). The cells that mediate DTH are T-lymphocytes. This reactions don't involve antibody or complement. These actions are referred to as "delayed' responses because, unlike reactions mediated by antibody that occurs MINUTES after antigen exposure (i.e.. ABO blood group incompatibility, hyper acute rejection, erythroblastosis fettles), delayed reaction occur one to two days following antigen exposure (this is why you need to wait 48 to 72 hours for your annual PPD test to be read). In DTH antigen is taken up by the dendritic cells and presented to to CD4 Th-lymphocytes on MHC Class II molecules. These stimulated Th-lymphocytes (usually of the TH1 lineage) then release interferon-g, which acts to stimulate and recruit macrophages leading to a monocytic infiltration of the area where the antigen is introduced. This is also responsible for the "walling-off" of M. tuberculosis infection in the lung and other forms of granulators inflammation with monicytic and giant cell infiltrates. b. B-lymphocytes are the major effector cells in the humoral immunity because they synthesize and secrete immunoglobulin. Hypersensitivity reactions mediated by antibodies include Type I (IgE mediated, i.e.. asthma, anaphylaxis), TYPE II (antibody mediated, i.e. ABO incompatibility hemolysis) and Type III (immune complex (ie. PSGN). B-lymphcoytes are stimulated by IL03 release from the TH2 subset of Th lymphocytes. c. Neutrophils are the primary phagocytes of the innate immune system and do not play a role in any of the four immune hypersensitivity reactions. Neutrophils are produced in the bone marrow and have a multi lobed nucleus. They are usually the first leukocytes to arrive at the site of inflammation and they are able to ingest and kill organisms by enzymatic and oxidative burst pathways. their presence in increased numbers in the blood is indicative of infection, especially with predominance of band forms referred to as a "left shift," but their numbers in the blood can be falsely elevated in a patient who has recently been administered corticosteroids. This effect of corticosteroids on the leukocyte count is attributable to the fact that corticosteroids cause" demmargination" (release from vascular walls) of these cells. d. eosinophils are phagocytic cells that are believed to play a role in the defense against parasitic organisms. These cells are present in small numbers in the blood stream and are believed to play a role in the pathogenesis of asthma as they are often found in increased numbers in the bronchial mucosa of patients with this illness, but asthma is primarily a Type I hypersensitivity response. e. Mast cells are granulocytes that exist in many tissues in the body and are primary mediators of the clinical effects of allergic reactions. They express Fc receptors for IgE, and IgE acts as the primary antigen receptor for these cells. When two membrane-bound molecules of IgE bind the same antigen, "cross-linking" occurs that directly leads to mast cell degranulation and release of mast cell granular contents such as histamine and heparin. Disorders of mast cells include urticaria pigments nd systemic mastocytosis.

544 A 3y.o boy has recurrent sinusitis and one episode of severe pneumonia. As part of his evaluation, Candidia extract is injected intradermally. After 48 hours later he returns with a firm nodule measuring 16 mm in diameter in the location where the extract was injected. Which of the following cell types is responsible for the response observed in this patient? a. T-lymphocytes b. B-lymph c. Neutrophils d. Eosinophils e. Mast cells

Hansen disease affects skin and nerves Etiology: Mycobacterium lapra. Transmission via respirator or prolonged skin to skin contact. Associated with the armadillo in the southwestern US. It likes low temperature which makes sense why the clinical manifestations occur in the skin, balls and superficial nerves. Path: depends on how strong Cell Mediated Immune responses are, which gives us two types of leprosy 1) tuberculoid (TL): least severe. patient has intact CMI that will launch a Th1-mediated response. Mild skin plaques will develop that are hypo pigmented. Hair follicle loss and focally decreased sensation. 2. lepromatous (LL): most severe. patient has weak CMI that will launch a Th2-mediated response. Macrophage signaling to kill M leprae is limited which will lead to: skin thickening, plaque like hypo pigmentation (often with hair loss), leonine facies, paresis, regional anesthesia, testicular destruction and blindness a. B. burgdorferi cause Lyme diseases. Erythema chronicum migrant with fever and malaise. Systemic disease can cause arthritis, facial paresis or cardiac involvement (eg, condition abnormalities). Prolonged untreated diseases will have CNS involvement. b. Campylobacter fetus, a gram - rod, causes mild enteritis in immunocompetent and mild systemic bacteremic illness in immunocompromised c. C. dish is a gram + rod. Clinical manifestations are often toxin mediated that can lead to lower respiratory tract infection. Complication can cause polyneuritis and myocarditis. e. T. palladium causes syphilis. -Primary: painlesss chancre -Secondary: diffuse erythamtous macule over the entire body including palms and soles and condyloma late formation -Tertiary: syphilis results in skin and bone gammas and ascending aortitits

A 34 y.o is a political refugee from East Africa. She has blotches of skin on her arms that have different color, and she also has a tingling sensation in her hands. PE shows patchy areas of skin anesthesia and hypo pigmentation on her upper extremities. Nerve biopsy shows many organism invading Schwann cells. HIV testing is neg. a. Borreilia burgdorferi b. Campylobacter fetus c. C. dish d. mycobacterium leprae e. treponema pallidum

CN XI is pure motor that passes through the posterior triangle of the neck that innervates the SCM and traps. Lesion will cause: -dooping of the shoulder -abduction of the arm above horizontal (d/t weakness in rotating glenoid upward) -winging of the scapula -If lesion is at the proximal portion of the nerve, weakness of the SCM can also be seen (cannot look away from the lesion) a. Deltoid (axillary nerve). Abduction of the arm at angles below the horizontal plane. b. Latissimus doors (thoracodorsal nerve). powerful adductor of the arm and assists with extension and medial rotation c. elevator scapulae (cervical and dorsal scapular nerve). elevate scapula and raise the medial border (inferior rotation of the glenoid). d. rhomboid major (dorsal scapular). draws scapular upward and medially e. serrates anterior (long thoracic). rotates the scapula upward, allowing abudction of the arm over the head. Similar presentation to traps injury, however it doesn't have shoulder drooping

11772 A 37 y.o previously healthy male has an enlarge lymph node in the posterior triangle of his neck. He got an excisions biopsy. Two weeks later, he has difficulty with overhead activities such as combing his hair or placing dishes on overhead shelves. PE shows a left shoulder droop with weakness of left arm abduction above the horizontal position. Other shoulder movements are normal, and there is no sensory loss. What muscle is messed up? a. deltoid b. lats c. elevator scapulae d. rhomboid major e. serrates anterior f. trapezius

Hepadnaviridae, mature HBV has HBcAg and HBsAg. -Genome is partially ds-circular-DNA -enter the nucleus, repair its dsDNA that serves as transcription into the intermediate +ssRNA -this +ssRNA serves as template for translation of viral proteins (eg, polymerase and reverse transcriptase) and reverse transcription into a ssDNA intermediate, that will be converted back into partially dsDNA

374 Describe the HBV genome replication in the liver cell.

Bronchial challenge testing is a highly sensitive but non-specific measure that can help exclude the diagnosis. Bronchial challenge testing assesses bronchial hyperactivity (BHR), a central pathophysiological feature of asthma BHR can be quantified as the concentration of an inhaled aerosolized bronchoconstrictive substance required to produce a 20% decline in FEV1. Methacholine, a cholinergic muscarinic agonist, is commonly used as the inhaled test substance. c. sputum eosinophilia is not sensitive or specific indicator of asthma. More for allergic asthma. d. IgE are generally elevated in patients with atopic (extrinsic allergic) asthma, patients with non-immune-mediated asthma have normal levels of serum IgE. Not sensitive/specific for intrinsic asthma.

663 A 14 y.o female has intermittent shortness of breath. FEV1/FVC ratio is 83%. Which of the following would be most useful in excluding asthma in this patient? a. normal chest x-ray b. negative methacholine challenge c. absence of eosinophilia d. normal serum IgE e. negative skin tests to various allergens

Adverse effects of succinylcholine 1. malignant hyperthermia (especially with halothane) in genetically susceptible patients 2. severe hyperkalmeia in patients with burns, myopathies, crush injuries and denervation 3. bradycardia from parasympathetic stimulation or tachycardia from sympathetic effects. Succinylcholine is a depolarizing NMJ blocking agent. Like Ach it attaches to the nicotinic acetylcholine receptor (nAChR) and depolarizes the euromuscular end plate. Unlike Ach, succinylcholine is not degraded by acetylcholinesterase, resulting in continuous stimulation of the endplate (reflected by intimal transient fasciculations). However, the Na channels surrounding the end plate rapidly become inactivated and cannot reopen until the end plate is depolarized. Thus, succinylcholine-induced depolarization remains isolated to the endplate, resulting in development of flaccid paralysis (phase 1 block). Eventually, with continued administration of succinylcholine, the continuous depolarization of the endplate gives way to gradual depolarization as the nAChR becomes desensitized to the effects of succinylcholine. This in termed phase II block and is similar to non-depolarizing block. Because the nAChR is a nonselective cation channel, its opening not only allows Na influx but also K release. Exaggerated hyperkalemia nd life-threatening arrhythmias can occur in patients with crush or burn injuries, denervating injuries or diseases (eg, quadriplegia and Guillain-Barre syndrome), and myopathies. These pathogloic states cause up regulation of muscle nAChRs and and/or rhabdomyolysis, which can result in the release of large amount son K when succinylcholine is administered. IN these patients, non-depolarizing agents such as vecuronium and rocuronium are better choices. a. atracurium is a nondepolazizing NMJ blocking agent releases histamine and can produce a fall in blood pressure, flushing and bronchoconstricition. It is also metabolized to laudanosine, which can provoke seizures. b. baclofen is a muscle relaxant that affects GABAb receptors at the level of the spinal cord c. dantrolene is a muscle relaxant effective in malignant hyperthermia. It acts on ryanodine receptors on the sarcoplasmic reticulum and prevents release of Calcium in to the cytoplasm of muscle fibers.

8481 A 45 y.o quadriplegic man has bacterial pneumonia is admitted to the hospital and started on IV antibiotics. Over the next 24 hours, he develops progressive respiratory failure requiring mechanical ventilation. Prior to intubation, a skeletal muscle relaxant is administered and the patient subsequently goes into cardiac arrest. His attached cardia monitor shows ventricular fibrillation, While he is being resuscitated, his serum potassium level is drawn and later comes back as 10.0 mEq/L. Administration of which of the following agent is most likely responsible for this patient's condition? a. atracurium b. baclofen c. dantrolene d. succinylcholine e. vecuronium

Keratins are a broad group of intermediate filaments present in epithelial cells. IM stained with keratin marker is used by pathologists to identify poorly differentiated tumors or tumors with an unclear site of origin. Keratin positive tumors are: -carcinomas, mesotheliomas, thymomas, various sarcomas, trophoblastic tumors and desmoplastic small round cell tumors. a. lymphocytes = CD3 b. muscle = actin, caldesmon, design d. endothelium = CD34, von Willebrand's factor e. Glial cells = GFAP (glial fibrillary acidic protein

1160 Dude has an enlarged left-sided supraclavicular lymph node. Node is painless and stony hard on palpation. Biopsy shows anapestic cells that stain positive for keratin. Which of the following is most likely the origin of this cell? a. lymphocytes b. epithelial surfaces c. muscle tissue d. endothelium e. glial cells.

Focal dystonia is a localized uncontrollable muscle contraction causing pain or discomfort as well as physical deformity in some cases. Local injection of botulinum toxin type B into the dystonic SCM results in muscular relaxation because toxin prevents presynaptic release of ACh, the NT responsible for muscle contraction, from the nerve terminal at the NMJ. This effect is temporary, however, because regeneration of the nerve terminal eventually occurs. (this process takes approximately three months.) For this reason, therapeutic botulinum toxin injections must be repeated when the effects begin to diminish. Botulinum toxin can also be used cosmetically to reduce the appearance of glabellar and other facial wrinkles. It is also used to relax the lower esophageal sphincter in esophageal achalasia, to treat the muscle spasms of MS and Parkinson's disease, and for other conditions resulting form involuntary muscle contraction. C. botulinum is a gram positive spore-forming anaerobic bacillus that synthesizes its point neurotoxins intracellularly and releases them by autolysis. a. antiphagocytic capsule is the primary virulence factor for S. pneumonia, H. influenza and Neisseria bacteria. b. hyper variable pili are characteristics of Neisseria meningitides and N. gonorrhoeae. c. S. aureus has an IgG-binding outer membrane protein, the Protein A virulence factor. Protein A binds to the Fc portions of IgG molecules thereby preventing opsonization, phagocytosis and complement fixation. d. intracellular polyphosphate granules are characteristics of C. diph. Granules within the cytoplasm are evident with methylene blue staining.

1401 There is a specific bacteria product that,when injected locally into the muscles of patients with relentless focal dystonia such as torticollis, produces a dramatic but temporary relief of symptoms. This substance is produced by bacteria that demonstrate: a. antiphagocytic capsule b. hyper variable pili c. IgG-binding outer membrane protein d. intracellular polyphosphate granules e. subterminal spore formation

Ingeneticially normal (46, XX) females, on eX chromosome is normally randomly deactivated each cell during early embryonic development. X-incativaiton (lionization) is maintained across cell division, resulting in clusters of cells throughout the body that express either the maternal or paternal X chromosome. this mosaic pattern of X-chromosome expression generally prevents X-linked recessive conditions from naifestating in female carriers. However, in rare cases, skewed lionization (uneven inactivation of maternal/paternal X chromosome d/t chance alone may result in females developing an X-linked recessive condition (eg, classic hemophilia). The process of lionization converts the inactive X chromosome into condense heterochromatin, which can be identified on microscopy as a compact by at the periphery of nucleus (Barr body). Heterochromatin consist of heavily methylated DNa (eg, cytosine converted to methyl cytosine) and deacetxyalted histones, which cause it to have a low level of transcriptional activity. A small proportion of genes rain transcriptionally active on the inactivated X chromosome. For this reason, inheritance of an abnormal number of X chromosomes can cause clinical manifestations d/t a gene dosage effect, as seen in Turner (45, XO) and Klinefelter (47, XXY) syndromes. a. during DNA replication, positive supercoiling occurs in the region ahead of the replication fork and must be removed for DNA replication to proceed. Topoisomerase's reduce DNA supercoiling by nicking the DNa strands, introducing negative coiling, and relegating the strands. b. double strand DNA breakage can occur following exposure to ionizing radiation. Compared to single strand breakage, double stranded breaks are more prone to result in faulty repair, leading to mutations, malignancy or cell death d. repair mismatched bases occurs throughout the genome during DNa replication. Impaired mismatch repair is associated with hereditary non polyposis colorectal cancer.

1486 A 24 y.o woman has a new mole on her right leg. She is worried that she might have skin cancer. PE shows a 5-mm brown, oval macule on her anterior thigh with a homogenous coloration and discrete borders. The lesion appears darker than her other moles. A biopsy of the lesion shows normal-appearing nevus cells clusters in the epidermis, and she is diagnosed with benign acquired melanocytic nevus. During histologic analysis, her epithelial cells are each found to contain a condensed body composed of heavily methylated DNA at the periphery of the nucleus. This regions most likely associated with which of the following genetic findings? a. DNA supercoil accumulation b. dsDNA break repair c. histone acetylation d. impaired mismatch repair e. low transcription activity

PAH is filtered from the blood in the glomerulus and screwed by the PCT. Because it is both filtered and secreted, it can be use to estimate RPF. Filtration cannot be saturated, however excretion mediated career enzyme mediated process that can be saturated. Therefore, as blood concentrate not PAH increases, the secretion of PAH by PCT increases, but only up to a maximum value of approximately 80 mg/min. Pass this point, secretion plateaus and any increases in the uric PAH concentration are d/t increased filtration. a. excretion of a substance is defined as = filtration + secretion - reabsorption. Once carrier mediated secretion and reabsorption mechanisms are saturated, these processes are at their maximal rater. However, filtration is not enzyme or protein mediated, so it doesn't show maximal value. Because filtration cannot be saturated neither can excretion. b. the maximal reabsorption rate doesn't apply to PAH, which is a secreted substance. Glucose is example of substance that is aggressively reabsorbed in the proximal tubule. Under normal conditions, no glucose is lost in the urine, despite the fact that glucose filtered into Bowman's space. In uncontrolled diabetes, however, glucose is seen in the urine because the reabsorption carrier proteins become saturated at very high glucose concentration d. filtration fraction is decreased by decrease in GFR or increase in RPH e. RPF is decreased by factors that constrict blood vessels in the kidney such as epinephrine, NE and angiotensin II

1619 It is estimated that for paraaminohippuric acid (PAH) the extraction ratio ((arterial plasma PAH - venous plasma PAH)/arterial plasma PAH) is near 90% of arterial plasma concentrations lower than 20. Once the plasma concentration of PAH increased above this level, the extraction ratio decreases progressively. Which of the following best explains the observed decrease in the PAH extraction ratio? a. maximal excretion rate is reached b. maximal reabsorption rate is reached c. carrier transport is saturated d. filtration fraction is decreased e. RPF is decreased

This patient has functional hypothalamic amenorrhea Pathophysiology: -reduced leptin d/t decreased adipose tissue -this inhibits pulsatile GnRH = decreased pit LH and FSH = low estrogen = amenorrhea. FHA can be caused by anorexia, thin female (caloric expenditure is out of proportion to intake). Complications -reduced peak bone mass = early onset osteoporosis

1839 A 19 y.o woman has amenorrhea. Her BMI is 18. Skin is thin and dry. What would her LH, FSH and estradiol like?

Patient has Wernicke aphasia (aka fluent aphasia): Wernicke-Word salad -located in posterior portion of the superior temporal gyrus in the dominant temporal lobe. -MCA supplies this area.

1846 A 54 y.o man cannot effectively communicate. He speaks clearly and with conviction but his stances are incomprehensible. He doesn't appear to understand the doctor's question, doesn't follow oral or written instructions, and cannot repeat simple phrases. What branch is occluded? a. ACA b. AICA c. MCA d. PCA e. PICA

The atherosclerotic occlusion of a coronary artery develops very slowly over time, there can be compensation by the development of arterial collaterals around the point of occlusion. The pressure gradient across the stenotic region facilitates blood flow through the small anastomotic vessels, which progressively dilate. The collateral circulation can provide flow to the hypo perfused areas distal to the point of occlusion. In this case, the patient likely developed collaterals from the right coronary artery to distal vessels normally perfused by the LAD a. when the fibrous cap overlying a coronary atherosclerotic plaque is thin, the plaque is unstable and predisposed to ulceration, fissuring, or rupture. Rupture of an atherosclerotic plaque can lead to overlying thrombosis and acute coronary occlusion, which resultant ischemic myocardial injury. This process occurs too quickly to be compensated by the development of new arterial collaterals. b. atherosclerotic rookery plaques with a rich lipid core are more prone to plaque rupture, and thus more lily to cause an acute ischemic coronary syndrome d/t superimposed thrombosis. c. activated macrophages in an atheroma contribute to collagen degradation by secreting metalloproteinases. When there is a high degree of intimal inflammation, release of these metalloproteinases is increased, which can destabilize the mechanical integrity of the plaque, Statins (HMG Co-A reductase inhibitors) decrease this inflammation and are useful in acute coronary syndrome to stabilize the plaque, e. the total coronary artery calcium content correlates modestly with the coronary artery atherosclerotic plaque burden. Thus, patients with advanced coronary calcification may be at increased risk fro ischemic myocardial injury. However, the calcium content of an individual plaque is not correlated with the probability of rupture or degree of plaque stability. Thus the presence of intra-plaque calcium could be consistent with either an acute contrary syndrome or with gradual occlusion without myocardial necrosis. f. An postal location of a coronary artery plaque would not give a finished tendency to cause ischemic myocardial injury. IN fact, it might promote such injury by compromising blood flow to a larger area of the myocardium. However a more important determinant of whether or not coronary plaque causes myocardial necoriss is the rate of arterial occlusion. A slowly progressive postal obstruction would not necessarily prevent the development of an adequate collateral circulation

39 Coronary angiography of a 69 y.o Caucasian female with chronic atypical chest pain shows extensive atherosclerosis and near total occlusion of the LAD artery. The absence of myocardial necrosis and scarring despite vessel occlusion in this patient can best be explained by which of the following features of the occluding plaque? a. thin fibrous cap b. rich lipid core c. active peripheral inflammation d. slow growth rate e. high calcium content f. ostial location

Risk factors: -preexisiting psychiatric disorders -hopelessness, impulsivity -*previous suicide attempts or threats* -divorced or separated -elderly white men -unemployed or unskilled -physical illness -FH of suicide -family discord -access to firearms -substance abuse Protective -social support/family connectedness -prgnancy -parenthood -religion and participation in religious acitivty

11618 Dude has a bunch of psychological illnesses. He is contemplating suicide. Which of the following is the strongest risk factor for sucide in this patient? a. age b. chronic pain c. FH of sucide d. martial status e. military service f. previous suicide attempt g. psychiatric illness h. sex

Celiac disease -Hypersensitivity to gluten. Gliadin, a breakdown product of gluten, triggers an immune mediate reaction causing: 1) Vilous atrophy 2) Crypt hyperplasia 3) lymphocyte infiltration Atrophy causes impairment of nutrient absorption in the duodenum and proximal jejunum, regions where concentration of gluten is the highest. Presentation: -presents 6-24 months after introduction of gluten with symptoms of malabsorption (eg, diarrhea, steatorrhea, flatulence, nutrient def and weight loss) -delayed puberty and short stature Screening: -IgA, anti-endomysial and anti-tissue transglutaminase Ab. Diagnosis: -endoscopic biopsy a. GI reflux is physiologic in healthy infants, Severe cases can lead to irritability, poor feeding and inadequate weight gain. Diagnosis is made clinically and treatment with conservative changes (eg, smaller feedings) and acid suppression (eg, ppi). b. Anti-inflammatory meds (eg, glucocorticoids and aminosalicylates) are the mainstay of treatment for inflammatory bowel disease (ulercative colitis and Crohns disease). Symptoms include abdominal pain, bloody diarrhea and weight loss. Colonoscopy shows diffuse colonic inflammation in UC and focal areas of small intestine and colonic inflammation in CD. c. Tropical sprue has similar histologic findings to celiac. No history of travel d/e: lactase intolerance has solar symptoms however histology would show normal small intestinal architecture

3224 A 14 month old girl has 2 month history of diarrhea. It was non bloody that is accompanied by occasional episodes of vomiting. She was breast fed until age 9 months and has since had a well varied diet including whole milk, fruits, vegetables, bread and meats. She has been less interested with food for the last several weeks. On PE, patient loss 2.5 lb in the last 2 months. Biopsy of the duodenum shows villous atrophy and crypt hyperplasia. what would improve her symptoms? a. acid reduction therapy b. anti-inflammatory meds c. antibiotic d. enzyme supplementation e. modified dairy diet f. modified grain diet

This patient has cerebrovascular and coronary atherosclerosis ( causing transient ischemic attacks and angina), which increases the risk of atherosclerosis in other organ systems. Atherosclerosis involving renal arteries can cause bilateral renal artery stenosis (RAS) in such patients. Reduced renal blood flow (eg, from dehydration or RAS) decreases GFR d/t fall in hydrostatic pressure. This stimulates renin production, which leads to angiotensin II formation. Angiotensin II causes systemic vasoconstriction with a resultant rise in BP. In the kidney, angiotensin II preferentially constricts the efferent arteriole. Blocked of this response by ACE inhibitors (ACEI) or angiotensin II receptor blockers (ARBs) causes the filtration pressure to fall, thereby reducing GFR. In unilateral RAS, the normal kidney compensates for decreased GFR< and overall creatinine clearance is maintained. However, patients with bilateral RAS may develop acute renal failure. Therefore, renal function should be monitored closely after starting gACEI/ARBs in patients with evidence of diffuse atherosclerosis. a/e: ACEI/ARBs have multiple cardiac benefits. They cause peripheral vasodilation that reduces after load. This decreases the contractile force needed to maintain CO, reducing myocardial oxygen demand. These drugs also slow the progression o left ventricular hypertrophy (in aortic regurgitation) and myocardial scar formation in expansion (eg, aneurysm formation after infarction). c/d: ACEI/ARBs may lead to hyperkalemia, which in turn can cause conduction abnormalities. However, in the absence of this side effect, ACEI have no direct effect on heart conduction. They can be used safely in patients with atrioventricular block or polymorphic ventricular extrasystoles f. angiotensin II is involved in atherogenesis. Hypertensive patients with iliofemoral atherosclerosis would therefore benefit form ACEI/ARBs that reduce the effects of angiotensin II.

695 A 75 y.o man has history of recurrent transient ischemic attacks, myocardial infarction 2 years ago and coronary artery bypass surgery for unstable angina 1 year ago. The physician considers adding ramipril to the patient's medication regimen to improve his HTN. Use of this medication is most likely to cause an adverse drug effect in patients with which of the following preexisting conditions? a. aortic regurgitation b. bilateral renal artery stenosis c. first-degree AV block d. polymorphic ventricular extrasystoles e. post-infarction left ventricular aneurysm f. severe iliofemoral occlusive disorder.

Patient has abetalipoproteinemia, a disease caused by the impaired formation of apolipoprotein B containing lipoprotein. Dietary lipids are normally processed in small-bowel enterocytes and secreted as chylomicrons; endogenously produced lipids are secreted from the liver hepatocytes as VLDL. Chylomicron and VLDL has apoB-48 and apoB-100, respectively. During the synthesis of api-b-containing lipoproteins, microsomal triglyceride transfer protein functions as a chaperone protein necessary for proper folding of apoB and also participates in the transfer of lipids to newly formed chylomicrons and VLDL particles. this is the most commonly caused by an AR, loss-of-funcito mutation int he MTP gene. It manifests during the first year of life with symptoms of malabsorption (eg, abdominal distention, foul-smelling stool). Lab shows very low plasma TAGs and cholesterol levels, and chylomicrons, VLDLs and apoB are entirely absent form the blood. Poor lipid absorption auses deficiency of fat soluble (particularly vitamin E) and essential fatty acids. This results in RBC with abnormal membrane and thorny projections called acanthocytes as well as multiple neurologic abnormalities (eg, progressive ataxia, retinitis pigmentosa) b. celiac biopsy will have atrophy and blunting of the villi in celiac disease. Chronic inflammatory infiltrate of the lamina proper is also present. c/e/g: no abnormalities are found in LM of the SI in chronic pancreatitis, Zollinger-Ellison syndrome or lactase def. d. LM of Crohn's shows chronic inflammation of all layers of the intestinal wall and noncaseating granulomas f. Whipple has distended macrophages in the lamina propria of the SI> Macrophages contain PAS-positive and diastase-resistant granules and rod-shaped T. whippelii bacilli.

327 A 5 m.o boy is being evaluated for poor weight gain since birth. He is not to have bulky and greasy stools. A jejunal biopsy shows the findings seen in a biopsy where the vili has a shit load of fats. This patent is most likely suffers from which of the following conditions? a. abetalipproteinemia b. celiac disease c. chronic pancreatitis d. crown's disease e. lactase def f. whipple disease g. zollinger elision syndrome

Kayser-Fleischer ring is most strongly associated with Wilson's disease (although can be seen in in chronic cholestatic diseases such as primary biliary cirrhosis). The rings are bored through the granular deposition of copper within Descent's membrane in the cornea. Pathology: AR disease identifiable in people from 5-40 y.o. The mutation hinder copper metabolism by reducing the formation and secretion of ceruloplasmin and decreasing hepatic secretion of copper into the biliary system. Copper is pro-oxidant thus will generate free radicals. It will eventually leak from injured hepatocytes into the circulation to be deposited in various tissues, including the cornea and basal ganglia. Atrophy of the basal ganglia then ensues. Treatment: focuses on life-long therapy of copper chelators such as d-penicillamine and trientine. a. pulmonary emphysema = alpha-1-antitrypsin def. b. DM develops in advanced hemochromatosis d. Renal cancer is from Hippel-Lindau disease e. hypertrophic cardiomyopathy is an AD disease involving the cardiac sarcomere. f. wilson doesn't cause congenital deafness.

398 Patient has Kayser Fleischer ring and chronic hepatitis. This patient is most likely to have which of the following conditions? a. pulmonary emphysema b. DM c. basal ganglia atrophy d. renal cancer e. hypertrophic cardiomyopathy f. congenital deafness

characteristics of pathogenic yeasts Candida (pseudohyphae w/blastoconidia) -parenteral nutrition (through a central venous catheter) -candida are susceptible to echinocandins. all are susceptible to fluconazole. Blastomyces (Broad based budding) -pulmonary infection (occasional dissemination) -unlikely cause of bloodstream infection in setting of catheter Coccidiodies (spherules w/endospores) -pneumonia, meningitis and osteomyelitis -endemic to southwestern US Cryptococcus (capsule india ink stain) -meningitis amount immunocompromised. Histoplasma (small oval yeast within macrophages) -subclinical. invasive in immunocompromised. endemic to Mississippi and Ohio river valleys e. Right angle branching hyphae= mucormycosis (mucorales or rhizomes). Doesn't grow in culture

11633 A 24 y.o woman has fevers and chills for the last 24 hrs. She had partial bowel resection for Crohn disease without complications. Since surgery, she has trouble eating and has peripherally inserted central venous catheter for parenteral nutrition. Her temp is 101, bp 100/60 and pulse 114. Abdomens oft and contender. Blood cultures shows yeast. Which of the following characteristic of the most likely organism causing this blood stream infection? a. broad base budding morphology b. dimorphic fungi endemic to southwestern US c. positive india ink stain d. pseudohyphae with blastoconidia e. right angle branching hyphae

Inclusion cell (I-cell) disease -AR lysosomal storage disease -defects in protein targeting, a process by which proteins are transported to their appropriate intra- or extracellular location. -Post translational mods (eg, folding, glycosylation, and phosphorylation) often function as markers that help guide the protein to its final destination. Proteins targets for lysosomes are mod differently than those destined for extracellular secretion. -Golgi body phosphotransferase enzyme catalyzes the phosphorylation of mannose residues on lysine bound proteins, allowing them to navigate through the Golgi network towards the lysosome. -defect in this enzyme leads to extracellular secretion of these proteins and accumulation of cellular debris int eh lysosome, forming the characteristic inclusion bodies.

11930 A 2 y.o has developmental delay and failed to thrive. He has recurrent ear infections since 6 months of age. PE shows coarse facial features, corneal clouding, hepatosplenomegaly and restricted joint mobility. Mass spec of fibroblasts reveals def phosphorylation of mannose residues on certain glycoproteins in the golgi apparatus. Normally, these proteins are most likely to be transported to which of the following cellular location? a. extracellular space b. lysosome c. mitochondria d. nucleus e. plasma membrane

Facial flushing and pupillary dilation = anticholinergic effects. Two main forms of cholinergic receptors: 1. nicotinic 2. muscarinic -toxic effects of anticholinergic meds affect mostly muscarinic receptors. -inhibitor of sweat gland = fever = compensatory cutaneous vasodilation -inhibing the pupillary constrictor and ciliary muscle = pupillary dilation Antihistamine is a class of drug that has anticholinergic effects a. even though he was treated with H1 blocker, action at this receptor doesn't account for his symptoms. H1 are found in the vascular endothelium and bronchial smooth muscle where they help mediate vascular permeability and bronchoconstriction, respectively. b. H2 blockers inhibits gastric acid secretion by parietal cells. d. nicotinic cholinergic receptors are found on preganglionic neurons in sympathetic and parasympathetic ganglia and on skeletal muscle cells at the neuromuscular junction. -Tubucurarine are commonly used as general anesthesia to induce paralysis. adverse effects are respiratory paralysis and autonomic ganglionic blockade can cause hypotension and tachycardia. e. alpha 1 adrenergic agonist helps allergic rhinitis by vasoconstricting to alleviate nasal congestion. Mydriasis will occur, but not flushing

1869 A 5 y.o boy is receiving treatment for allergic rhinitis has flushed cheeks and dilated pupils on PE. These findings are best explained by: a. H1 blocker b. H2 blocker c. antimuscarinic d. anti nicotinic e alpha1-adrenoreceptors agonist

Patient has infective endocarditis = constitutional (flu like) symptoms, over and new systolic murmur. Elevated serum creatinine with hematuria and proteinuria = renal insufficiency from nephritic syndrome. IE may be complicated by deposition of circulating immune complex in the glomerular capillary wall, resulting in glomerulonephritis. This can be seen as capillary wall thickening with sub endothelial and sub epithelial deposit formation. a. anti-glomerular basement membrane autoantibodies = good pasture syndrome. Causes acute nephritic syndrome but not cardiac symptoms. c. IE is rarely caused by gram neg organism, the producers of endotoxin. Furthermore, endotoxin causes acute tubular necrosis d/e: emboli from infected endocardial vegetations may metastasize via the hematogenous route to cause infarcts in the brain, kidneys, myocardium and other tissues (eg, Laneway lesions on palms and soles). Emboli are usually small and do not cause acute renal failure. Renal failure may be seen if the infectious embolic focus develops into an abscess or the embolus is large enough to cause sizable infarct. However,r both renal infarct and abscess present with flank pain.

229 A 23 y.o man has 3 week history malaise and fatigue. He says he has been sick with the flu for the last 3 weeks. Cardiac auscultation reveals an apical holosystolic murmur radiating to the axilla, which was not heard during previous office visits. Lab shows elevated creatinine, mild proteinuria and microscopic hematuria with red cell casts upon urinalysis. Which of the following is the most likely pathogenesis of this patient's renal findings? a. antiglomerular baseemembrane antibodies b. circulating immune complex mediated injury c. endotoxin induced renal tubular injury d. hematogenous metastatic infection focus e. thromboembolic event

Patient has high systolic blood pressure and normal diastolic blood pressure = isolated systolic hypertension. Aging is associated with increased arterial stiffness caused by endothelial dysfunction and a change in extracellular matrix compassion (eg, decreased elastin, increased collagen deposition). Increased arterial stiffness leads to decreased compliance of the aorta and major peripheral arteries, causing elevated pressures during systole. ISH can also result from increase in CO d/t severe aortic regurgitation or systemic causes (eg, anemia, hyperthyroidism). b. primary decrease in CO would lower systolic, diastolic and MAP. c. decrease pulmonary residual volume with age d/t destruction of alveolar walls, which can be associated with minor increase in pulmonary vascular resistance. However, this would not have a significant effect on arterial pressures in the systemic circulation. d. increase in sympathetic tone leads to increase in peripheral vascular resistance would be expected to increase SBP and DBP. This plays an important role in pathogenesis of HTN in patients younger than 50. e. renal artery stenosis = renovascular HTN = activation of RAAS = hypervolemia and increased in total peripheral vascular resistance = increase in both SBP and DBP

456 A 78 y.o man has periodic headaches relieved by tylenol. BP 180/70. PE shows fourth heart sound but otherwise unremarkable. Never smoked. CT scan of head is unremarkable Which of the following age-related changes best explains this patient's blood pressure reading? a. aortic stiffening b. decrease CO c. decreed lung residual volume d. increase in sympathetic tone e. renal artery atherosclerosis

teratogenic meds Phenytoin: neural tube defects, orofacial clefts, microcephaly, nail or digit hypoplasia Lithium: Ebstin anomaly, nephrogenic diabetes insidious, hypothyroidism Valproate: neural tube defects Isotretinoin: microcephaly, thyme hypoplasia, small ears, hydrocephalus Methotrexate: limb and craniofacial abnormalities, neural tube defects, abortion ACE inhibitor: renal dysgenesis, oligohydraminos Warfarin: nasal hypoplasia, stippled epiphysis Valproate treats -migraine -epilepsy -bipolar Valproate can interfere with folate metabolism and increase fetus chance of having neural tube defects. Use levetiracetam instead. Give high dose of folate. a. First trimester exposure to warfarin can lead to warfarin embryopathy characterized by depression of the nasal bridge and stippling (pinpoint calcification) of the epiphysis. It can also cause nail hypoplasia and fetal bleeding. b/g: Lithium can cause hypothyroidism and Ebstein anomaly. Neonatal hypothyroidism can also be a result from exposure to maternal antithyroid drugs (eg, propylthyiouracil, methimazole) d. phenytoin is an anti epileptic that can cause orofacial clefts, microcephaly, nail or digit hypoplasia, cardiac defects and dysmorphic facial features. e. renal dysgenesis and fetal anuria/oligohydraminos is caused by ACE inhibitors and angiotensin receptor blockers. f. short palpebral fissure, smooth philtrum and thin vermillion border are key dysmorphic features of fetal alcohol syndrome.

507 Patient is taking valproate and labetalol. Her offspring has an increased risk of having what? a. epiphyseal stippling b. hypothyroidism c. myelomeningocele d. nail hypoplasia e. renal dysgenesis f. short palpebral fissures g. tricuspid valve malformation

SLE is type III hypersensitivity. Immune complex mediated tissue injury in various organs. CVS: accelerated atherosclerosis with increased risk of MI. Small vessels can show evidence of vasculitis with fibrinoid necrosis in any tissue. other cardiac features such as pericarditis and less commonly verrucous (Libman-Sacks) endocarditis, characterized by thickened valve leaflets with multiple small vegetations on both surfaces. The vegetations are composed of sterile platelet thrombi intertwined withs trans of fibrin, immune complexes, and mononuclear cells. these vegetations are easily dislodged and can result in systemic embolization Renal: diffuse proliferative glomerulonephritis characterized by proliferative and necrotizing lesions with crescent formation during active disease. LM shows diffuse thickening of the gloemrualr capillary walls with "wire-loop" structure d/t sub endothelial immune complex deposition.

73 A 38 y.o woman suddenly collapses at home after experiencing severe chest pain and dies before reaching hospital. Postmortem examination shows thrombus overlying a ruptures atherosclerotic plaque on the LAD. The patient is also found to have thickened mitral valve leaflets with multiple small vegetations on both valvular surfaces and fibrinoid necrosis of arterioles. Other findings include glomerular capillary basement membrane thickening with wire-loop changes. this patient was most likely suffering form what? a. bacterial endocarditis b. churg-strauss syndrome c. granulomatosis with polyangiitis d. SLE e. thromboangiitis obliteran

Etanercept is a TNF-alpha inhibitor added to methotrexate to treat moderate to severe RA. Acts as a decoy receptor. Pharmaceutical companies provide the prefix of the names for the biological agents. The suffix indicates whether the med is -monoclonal antibody (mab) -receptor molecule (kept) -knase inhibitor (nib) a. rituximab is a chimeric monoclonal antibody targeted against CD20, a cell receptor found on the surface of B cells. It is used to treat CD20, non-Hodgkin's lymphomas and other diseases related to excessive B cell function (tu for tumor; mab for antibody) b. Infliximab is a chimeric monoclonal antibody targeted against TNF-alpha. It is used in treatment of a number of autoimmune disease (eg. RA and crowns). li = immunomodulating/xi=chimeric (human constant regions/foreign variable regions) c. cetrolizumab is a humanized monoclonal antibody that targets TNF-alpha. It lacks Fc region (preventing complement activation and cell mediated cytotoxicity) and is used in treating autoimmune disorders associated with elevated levels of TNF-alpha. li = immunomodulating. zu = humanized (human constant and variable regions, except complement determining regions) d. imatinib mesylate use to treat specific cancers, including Philadelphia chromosome-positive chronic myelogenous leukemia and kit-positive GI tumors. It is an example of a small molecule tyrosine kinase receptor inhibitor.

8523 Patient has rheumatoid arthritis. Despite treatment, she continues to have several hours of morning stiffness daily and frequently awakens at night d/t joint pain. PE shows swelling and tenderness in the joints of her hands and wrists. The med is best characterized as which of the following? a. cell surface receptor antibody b. chimeric monoclonal antibody c. humanized monoclonal antibody d. small molecule receptor inhibitor e. soluble receptor decoy protein

PCA supply th e occipital lobe, which contains the striate or primary visual cortex. The striate cortex of each hemisphere receives info about contralateral visual field from the ipsilateral geniculate nucleus via the optic radiation. -cuneus gyrus = lower visual field/upper retina -lingual gyrus = upper visual field/lower retina Occlusion of PCA will cause contralateral homonymous hemianopia with macular sparing. The macula is spared d/t MCA a. anterior cerebral artery occlusion = contralateral weakness that predominantly affects the lower extremity. b. occlusion of the central retinal artery leads to sudden, painless, and complete loss of vision c. MCA lesion is characterized by contralateral motor and sensory deficits (more pronounced in the upper than lower limb) and homonymous hemianopia with macular involvement e. PICA occlusion leads to lateral medullary (Wallenberg) syndrome. it is characterized by contralateral loss of pain and temp sensation; ipsilateral def of CN V, VIII, IX, X and XI and Horner's syndrome.

8592 Patient has right homonymous hemianopia with macular sparing. Which vessel is occluded? a. anterior cerebral b. central retinal c. middle cerebral d. posterior cerebral e. posterior inferior cerebellar

Sensitivity = TP/(TP + FN). test to identify those with given disease. Rule in. Specificity = TN/(TN + FP). test to exclude those from having a given disease. Rule out. Moving cutoff value to the right would increase specificity at the expense of sensivity, While moving cutoff value to the left would increase sensitivity at the expense of specificity.

1285 sensitivity vs. specificity

Patient has stable angina (angina pectoris) d/t underlying CHD. Aspirin irreversibly inhibits COX. Inhibiting COX1 in palettes prevent synthesis of thromboxane A2, a potent stimulator of platelet aggregation and vasoconstriction = reduce the risk of occlusive thrombus formation and subsequent MI. Some patients are unable to tolerate aspirin d/t exacerbation of preexisting respiratory symptoms (eg, rhinos and asthma) or delveopment of allergy reactions (eg, urticaria, angioedema, anaphylaxis). A good alternative for these patients is Clopidogrel: MOA: blocks the P2Y12 component of ADP receptors on the platelet surface and prevent platelet aggregation. -it is as effective as aspirin for prevention of cardiovascular events and should be used in patents with aspirin allergy. a/g. Apixaban is direct factor Xa inhibitor that prevents platelet activation and fibrin clot formation. Warfarin inhibits enzyme required for synthesis of active vitamin K, resulting in decreased synthesis of vitamin K-dependent clotting factors (II, VII, IX and X). Both of these meds are typically used for prevention and/or treatment of thromboembolic events, not CAD. b. cilostazol is a phosphodiesterase inhibitor that occasionally used in patients with symptomatic peripheral vascular disease (i.e., claudication) d. low molecular weight heparins (LMWHs; enoxaparin, dalteparin) are indirect thrombin inhibitors that bind with antithrombin and convert it form a slow to a rapid activator of thrombin and factor Xa. They are used in patients with acute coronary syndrome (unstable angina or MI) but have no role in the management with stable angian. e. eptifibatide is a platelet glycoprotein IIb/IIIa inhibitor that inhibits the final common pathway of platelet aggregation. It is occasionally used in some patients with acute coronary syndrome, but not for stable CHD. f. naproxen is an NSAID used for pain and management in patients with osteoarthritis. Use of both nonselective and selective COX2 NSAIDSs increases the risk adverse CVS event.s

1828 A 64 y.o dude is having exertion chest pain for the past 6 months. He smokes a shit lot. Has type 2 dm and peripheral artery disease. Patient undergoes treadmill exercise stress testing and develops substernal chest pain on moderate exertion accompanied by ECG changes that resolve immediately upon rest. e was given a low dose of aspirin therapy for secondary prevention of cardiovascular disease but experiences sob and wheezing with the medication. Which of the followings the best alternate therapy for this patient? a. apixaban b. cilostazol c. clopidogrel d. enoxaparin e. eptifibatide f. naproxen g. warfarrin

Hepatic processing of bilirubin 1. carrier mediated uptake of bilirubin at the sinusoidal membrane 2. conjugation via UGT (urdine diphosphate-clucuronyltransferase) in ER 3. Biliary excretion of water-soluble, nontoxic bilirubin glucuronides. Disruptions of these processes can lead to Crigler-Najjar syndrome. -Type 1: AR. Lack of UGT enzyme. Unconjugated hyperbilirubinemia will develop. Conjugated bilirubin is water soluble and loosely bound to albumin = easy excretion Unconjugated bilirubin are less soluble and binds tightly to albumin = not excreted, that will graduallydeposited into various tissues, including the brain. Kernicterus (bilirubin encephalopathy), can be fatal. b. Dubin-Johnson is AR. Absence of biliary transport protein, MRP2 (multi drug resistance protein 2), used in the hepatocellular excretion of bilirubin glucuronides into bile canaliculi. Liver is darkly pigmented, however disease is asymptomatic c. rotor syndrome is rare AR d/t asymptomatic conjugated hyperbilirubinemia that results from numerous defects in the hepatic uptake and excretion of bilirubin pigments. Although patients are often jaundiced, they enjoy normal life expectancies. d. High amount of anaerobes and S. aureus can deconjugate bile acids (via removal of glycine and taurine), rendering them less soluble and therefore less able to form micelles. This beconjugation impedes the active reabsorption of bile acids into portal circulation at the terminal ileum, resulting in lipid malabsorption. However, there are no neurologic abnormalities associated with bilirubin beconjugation in the gut e. in the colon, bacterial enzymes reduce bilirubin into urobilinogens, where a small fraction returns to the liver where they will later either re-enter GIT or excreted via urine. Urobilinogens that remain in the colon are excreted in the feces as stercobilin, contributing to its dark color. Fortunately, there are no neurologic abnormalities associated with impaired gut reabsorption of bilirubin.

363 A one month old white kid is having history of persistent jaundice experiences muscle rigidity, lethargy and seizures. Which of the following causes of hyperbilirubinema is most likely to produce this patient's neurological abnormalities? a. absent liver conjugation enzymes b. deficient bilirubin excretion into bile canaliculi c. impaired canalicular bile transit d. increased gut beconjugation of bilirubin e. impaired gut reabsorption of bilirubin

PSGN is a nephritic syndrome (eg, periorbital edema, hematuria, HTN, red cell casts and mild proteinuria). Labs: -decreased C3 -elevate titers of streptococcal antibodies (eg. anti-DNAse B, anti-hyaluronidase, antistreptolysin O (aka ASO, no detectable with impetigo) Imaging: -LM: hyper cellular glomeruli (from leukocyte infiltration (eg, neutrophils and monocytes)) and mesangial and endothelial cell proliferation. -IM: granular deposits of IgG, IgM and C3 (showing a "starry sky" appearance). -EM: sup epithelial humps (from deposition of antigen-antibody complexes). b. PSGN activates the alternative/lectin complement pathway, resulting in glomerular C3 deposition without significant C1 or C4 deposits. -Sub endothelial C1q deposits are characteristic of type 1 membranoproliferazive GN. d. fibrin is found in rapidly proliferative (crescentic) GN e. IgE is sometimes seen in lupus nephritis and are confined to the capillary wall. f. M protein is a component of the streptococcal cell wall that acts as an antiphagocytic virulence factor. PSGN is caused by nephritogenic antigens such as sure. pyogenic exotoxin B, not M.

What are the florescent areas on an IM of PSGN contain? a. albumin b. C1q c. C3b d. fibrin e. IgE f. M protein

This patient is given GH. MOA: binds to cell surface receptors that stimulate the production of IGF-1, primarily located in the liver. -Once bound to 2 receptors on the cell surface, it activates Janus Kinase (JAK), a non receptor tyrosine kinase. -JAK phosphorylates several tyrosine residues in the intracellular domain of GH receptor -Phosphorylation of tyrosine serves as a docking sites for STAT (single transducer and activator of transcription), where it will be phosphorylated also by JAK. -STAT then dimerizes and translocates to the nucleus to induce IGF-1 gene transcription by binding to the promoter region. a. steroids and thyroid hormones bind to intracellular receptors. b. several hormones (eg, PTH, ACTH, TSH & ADH (V2)) activates cAMP/PKA. c/d: various hormones (eg, GnRH, TRH, angiotensin II, ADH (V1)) function by binding to the DAG/IP3/PKC pathway.

1720 Patient has Turner Syndrome (45 XO). She is given medication to improve growth to normalize her height. Which of the following intracellular pathways is stimulated by the meds? a. binding to activated receptors to DNA to modify transcription b. cAMP/PKA pathway c. DAG/PKC d. IP3/Ca/Calmodulin e. JAK-STAT

Gonococcal cervicitis -clinical features: 1. purulent or mucopurlent discharge. 2. Friable cerbix with easy bleeding (eg, inter menstrual or postcoital bleeding) 3. mucopurulent discharge or inter menstrual or post coital spotting -diagnosis: PCR -Empiric treatment: 3rd generation cephalosporin (ceftriaxone) and azithromycin or doxycycline (for Chlamydia) Complications: PID = scarring of fallopian tubes from salpingitis that can cause infertility and increase chance of ectopic pregnancies. a/b. gonococcal and chlamydial infections are not risk factors for cervical cancer. Also, family history is also not a risk factor. HPV 16 and 18. c. endometrial hyperplasia presents as dysfunctional uterine bleeding d/t elevated estrogen without opposition by progesterone leading to over stimulation of growth to the uterine lining. Obesity is a risk factor for endometrial hyperplasia d/t excess aromatization of androgens to estrogen in the adipose tissue. e. use of cominbaiton oral contraceptive pills decreases ovarian cancer risk. Cervicitis is not a risk factor for ovarian cancer.

1008 A 24 .o obese woman has spotting after vaginal intercourse. She has been taking contraceptive pills for the past 3 years and no other meds. FH of cervical cancer. Her BMI is 35. Speculum examination reveals purulent discharge from the cervical os. The cervix is friable. There is no cervical motion tenderness on bimanual examination and the adnexa are contender. Microscopy of discharge demonstrates abundant neutrophils and PCR testing is positive. If left untreated, the patient's condition could lead to which of the following complications? a. cervical cancer b. cervical insufficiency c. endometrial hyperplasia d. infertility e. ovarian cancer

Isolation of the fastidious N. gonorrhoeae requires a selective medium the twill inhibit the growth of the other bacteria normally present in the oral cavity. A chocolate agar-based medium containing various antibiotics, the Thayer-Martin medium (and modified Thayer-Martin medium) is commonly used to isolate Neisseria from clinical specimens. The commonly used antibiotics are: -vancomycin to inhibit gram-positive bacteria -colistin (i.e., polymyxin) to inhibit gram negative bacteria (commensal Tessera species but not N. meningitides or N. gonorrhea) -Nystatin to inhibit yeast -TMP to inhibit the Proteus species a. E. coli is gram-neg rod which is inhibited by colistin (i.e., polymyxin) and TMP. In addition, it is not associated with pharyngitis however it is the most common causes of gram neg sepsis b. S. pyogenes (i.e., group A strep_ is an important pharyngeal pathogen because it can cause pharyngitis that if left untreated, can lead to rheumatic fever. Strep is gram positive is being taken care of by Vancomycin in the Thayer-Martin selective culture C. S. pneumoniae is being inhibited by vancomycin. It is not an important cause of pharyngitis but it is an important cause of meningitis and community-acquired pneumonia in adults. d. K. pneumoniae is gram neg rod that is being taken care of by colistin (ie. polymyxin) and TMP. It is not associated with pharyngitis but classically causes severe pneumonia in alcoholic or otherwise severely disabled patients. It also causes nosocomial sepsis and UTIs. It is a part of the oropharyngeal normal flora and characteristically causes pulmonary infection following aspiration. f. C. diph was historically important causes of pharyngitis in the US prior to the introduction of the diphtheria-pertussis-tetanus vaccine. Children clinically present with fever and sore throat, with an evident pseudomembrane. It is a gram positive organism inhibited by vancomycin.

1024 Pharyngeal educate sample from a 22 y.o woman is being placed on selective medium containing vancomycin, colistin, nystatin and trimethoprim. This medium favors growth of which of the following bacteria? a. E. coli b. S. pyogenes c. S. pneumoniae d. K. penuomoniae e. N. gonorrhoeae f. C. diph

VZV (enveloped dsDNA) Initial infection is called varicella (chicken box) Then goes dormant in DRG or trigeminal ganglia Reactivation is called zoster (shingles) Presentation: unilateral burning pain and papular or vesicular rash in a dermatomal distribution. The lesions coalesce, rupture, crust over and health within a few weeks although the discomfort can last longer. Investigations: -Intranuclear inclusion in keratinocytes -Multinucleated giant cells (Tzanck smear, can also be seen in Herpes) -Skin biopsy shows pacantholysis (loss of intercellular connections) of keratinocytes and intraepidermal vesicles. a. pacantholysis forming supra basal blisters = Pemphigus Vulgaris (PV). IF shows deposition of IgG containing deposits in a reticular pattern around keratinocytes. Immune target is the desmosome protein desmoglein 3. b. Molluscum contagious (a poxvirus) causes eosniphilic cytoplasmic inclusions (aka molluscum bodies) in infected cells and dome-shaped, umbilicate papule on examination c. HPV causes warts. LM revels cytoplasmic vacuoles in keratinocytes (koilocytosis) and hyperplasia of the epidermis d. dermatitis herpetiforims presents with pruritic, grouped vesicles on extensor surfaces. LM shows accumulations of neutrophils on the tips of dermal papillae (micro abscesses).

1042 A 64 y.o man has 4 day history of severe left sided chest discomfort. The pain is constant and has a burning quality. He seems febrile but everything else is normal. PE shows a vesicular rash at the 5th left intercostal space. Which of the following things does he have? a. acantholysis and intercellular IgG deposits b. eosinophilic cytoplasmic inclusion c. koilocytosis of the superficial epidermal layers d. micro abscesses at the tips of dermal papillae e. multinucleate cells with intranuclear inclusions.

The unit in this study is population not individuals. This is consistent with an ecological study, in which the frequency of a given characteristic (eg, vitamin D intake) and a given outcome (eg, multiple sclerosis (MS)) are studied using population date. Ecological studies are useful to generate hypotheses but should not be used to make conclusion regarding individuals within these populations. a/b: case control and cohort studies would start with individuals rather than populations. -In case-control studies, the odds of exposure to a certain characteristic (eg, high or low vitamin D intake) is compared between affected individuals (eg, patients with MS) and unaffected individuals who serve as controls. -In cohort studies, individuals (with and without different exposures such as high or low vitamin D intake) are followed over time to determine incidence of disease of interest (such as MS). c: cross-sectional surveys would also evaluate the exposures and outcomes of interest in individuals (not populations) at a given point in time ("snapshot"). e. nested case control designs start with cohort studies in which participants are followed over time,a nd those participants who develop an outcome of interest become cases for a case-control study. f. qualitative studies are focused discussion groups, interviews (structured and semi-structured), and other anthropologic techniques to obtain narrative information that can be crucial in explaining quantitative results. g. randomized controlled trials enroll individuals who will be randomly assigned into a treatment group or a control group. the groups differ only in terms of intervention (treatment) of interest). h. systematic reviews and meta-analyses take several studies (with an emphasis on high-quality randomized controlled studies) and attempt to group the results to obtain a pooled effect estimate .

10570 A large multi-country study used population data from 14 countries located within similar latitudes to evaluate the association between dietary habits, including vitamin D intake, and prevalence of MS. Which of the following best describes the design of this study? a. case-control study b. cohort study c. cross sectional survey d. ecological study e. nested case control study f. qualitative study g. randomized controlled trial h. systematic review

Heparin is the most important cause of thrombocytopenia in hospitalized patients. Occurrence of HIT (heparin induced thrombocytopenia) is much more common with use of unfractonated heparin compared to low molecular weight heparin. HIT more commonly leads to paradoxical thrombosis rather than bleeding. HIT is a serious disorder caused by antibodies to heparin and platelet factor IV. Direct thrombin inhibitor (huridin, lepirudin and argatroban) do not require antithrombin-III for their actions and are drugs of choice in treatment of HIT. Patients with HIT need ongoing anticoagulation d/t the presence of, or possibility of thrombosis. Upon clinical suspicion of HIT, the most important initial step in treatment is to stop all forms of heparin. b. Warfarin inhibits vitamin K dependent y-carboxylation of glutamic acid residues of clotting factors II, VII, IX and X. c/d/e: platelet plug formation involves there critical steps: adhesion, aggregation and release. Drugs that inhibit platelet aggregation work by different mechanism. -Aspirin irreversibly acetylates platelet COX-1 leading to decreased formation of thromboxane. This drug is sued for primary and secondary prevention of MI and strokes -Ticlopidine and clopidogrel inhibit ADP mediated platelet aggregation. They are useful following percutaneous coronary intervention (PCI), and for treatment of unstable angina and non-Q wave MI. -Dipyridamole and cilostazol inhibit platelet aggregation by inhibits phosphodiesterase activity and increasing cAMP. -Glycoprotein IIb?IIIa inhibitors inhibit biding of platelet glycoprotein IIb/IIIa with fibrinogen and fibronectin. The glycoprotein IIb/IIIa inhibitors abciximab, eptifibatide and tirofiban are approved for use following percutaneous interveion (PCI) in acute coronary syndrome.

1078 A 55 y.o man has unstable angina. On the fourth day of hospitalization he develops severe foot pain and right toe paleness. His lab testing is significant from thrombocytopenia. Treatment of argatroban is initiated immediately. What is this drug's MOA? a. binds to thrombin active site b. blocks glutamate residue carboxylation c. block ADP receptors d. blocks GP IIb/IIIa surface receptors e. inhibits phosphodiesterase

This patient has appendicitis, which can cause both visceral (dull,non-localized) and somatic (severe, well-localized) abdominal pain. Visceral abdominal pain is most often d/t luminal distention and stretching of smooth muscle and is carried by general visceral afferent fibers of the autonomic nervous system. The pain typically occurs int he midline region and is poorly localized and of a dull, constant, or cramping quality. Patients can also develop nausea, vomiting, or sweating d/t the autonomic stimulation. Somatic pain is usually d/t irritation of the parietal peritoneum and is well localized, more severe, and worsened with deep inspiration or pushing on the abdominal wall. the afferent pain fires for the appendix , proximal colon and overlying visceral peritoneum cross through the superior mesenteric plexus and enter the spinal cord at the t10 level to produce vague, referred pain at the umbilicus. As the appendix becomes more inflamed, it irritates the parietal peritoneum and abdominal wall and causes a more severe somatic pain that shifts from the umbilical region to McBurney point (two-third of the distance from the umbilicus to the anterior superior iliac spine). The abdominal wall becomes very sensitive to gentle pressure or sudden release of deep pressure (rebound tenderness). b/c: the appendix is usually 2 cm beneath the ileocecal valve in the RLQ. Depending on its orientation, there can be additional clinical findings. An inflamed appendix lying against the right obturator interns muscle can cause right lower quadrant pain with internal rotation of the right hip. If the inflamed appendix lies against the right tpsoas muscle, there can be pain with hip extension. d. the cecum is innervated by the superior mesenteric plexus and has only visceral sensation. e. patients with retrocelcal appendix may not have significant right lower quadrant tenderness,as the inflamed appendix doesn't contact the anterior parietal peritoneum and the cecum (distended with gas) acts as a cushion that blocks the examiner's hand.

11775 A 21 y.o man comes to the emergency room d/t abdominal pain, nausea, and vomiting. The patient started having dull periumbilical pain several hours ago, but the pain is now more severe and localized to the right lower abdominal quadrant. The patient keeps his hips flexed and there is maximal tenderness one-third of the distance from the anterior superior iliac spine to the umbilicus in the right lower abdomen. Bowel sounds are decreased. Lab shows leukocyte count of 16k. Which of the following is the most likely cause of the change in the quality of pain this patient? a. inflammation of the parietal peritoneum. b. involvement of the obturator interns muscle c. involvement of the psoas major muscle d. irritation of the cecal nerve endings e. retrocecal orientation of the appendix

Medications associated with osteoporotic fractures and possible mechanism -anticonvulsants that induce CYP450 (phenobarbital, phyntoin and carbamazepine) via increase vitamin d catabolism -aromatase inhibitors and medroxyprogesterone decrease estrogen -GnRH agonist decrease testosterone and estrogen -PPIs decreases calcium absorption -Glucocorticoids, unfractionted heparin and thiazolidinediones all decrease bone formation Insoluble calcium (eg, calcium carbonate) requires an acidic environment for proper absorption and acid suppressing medications can interfere with the process. This can be particularly problematic in older adults who already have reduced calcium absorption already. abcd: all have beneficial effect on bone metabolism (statins, thiazide, beta blocker and nitrates) f. anticonvulsants that induce hepatic P450 (phenytoin, carbamazepine and phenobarbital, primidone) may increase vitamin d catabolism and elevated PTH levels leading to increase done turnover and higher risk of osteoporosis. However, topiratmate doesn't induce P450 enzyme and has not been shown to affect bone mineral density in adults.

10930 Long term use of which of the follow meds will increase a patient osteoporosis and hip fracture? a. atorvastatin b. chlorthalidone c. metoprolol d. nitroglycerin e. omeprazole f. topiramate

EHEC (O157:H7) can cause hemorrhagic colitis. In contrast to most E. coli strain it doesn't: -ferment sorbitol -producse glucuronidase Toxin is shiga like that binds to the 60s ribosomal subunit. Complications: HUS a. toxins that activates cAMP are from Bordetalla pertussis, ETEC (LT), C. jejuni enterotoxin and cholera toxins. This leads to decreased absorption and increased secretion of sodium, chloride and water. b. toxins that activates cGMP are ETEC (ST) and Yersinia enterocolitica leading to watery diarrhea and loss of electrolyte. c. C. diff produces an exotoxin capable of disrupting the cytoskeleton (toxin B/cytotoxin). This toxin acts by depolymerizing actin, leading to cell death.

1100 How does EHEC toxin work? a. activates adenylate cyclase b. activates guanylate cyclase c. disrupts the cellular cytoskeleton d inactivated EF-2 e. inactivates ribosomal subunits

LPS is related during destruction of bacterial cell wall. It is long, heat stable molecule arranged into three regions: O antigen, core polysaccharide and lipid A, the last is responsible for toxic properties of LPS that can lead to Gram neg sepsis and endotoxic septic shock. Lipid A induces shock by activation of macrophages and granulocytes. -Leading to synthesis of endogenous pyrogens, such as IL-1, prostaglandins and inflammatory mediators (TNF-alpah and interferon). -Tehse cytokines then induce a febrile response by the action of IL-1 on the hypothalamus, as well as hypotension, increased vascular permeability with third spacing of fluid, diarrhea, DIC and death. a. E. coli strains that causeneonatab meningitis synthesize K-1 capsular antigens b. heat stable exotoxin is one of the enterotoxins produced by ETEC c. O antigen is cell wall outer membrane polysaccharide antigen used to classify gram neg bas d. fimbriae or pili are virulence factor that allow banter to adhere to target tissue and establish infection f. Flagellar (H) antigen is a heat labile protein which is one component of the serologic classification of the enterobacteriaceae.

1141 Dude is having septic shock d/t E. coli infection. What is causing this? a. capsule b. heat stable exotoxin c. O antigen d. Fibril antigen e. lipid a f. flagellar antigen

CN V: exits brain stem at the ventrolateral pons. Injury would lead to loss of: -Ipsilateral facial sensation -conreal reflex (afferent limb) -paralysis of muscle of mastications CN VI: exits brain stem from the pontomedullary junction medial to the facial nerve. Lesion leads to: -inability to abduct ipsilateral eye -horizontal diplopia (lack of innervation to the lateral recutus) CN VII: exits the brain stem from the ventrolateral pontomedullary junction, just lateral to the abducens. Damage would cause: -ipsilateral facial muscle paralysis (upper and lower) -Loss of corneal reflex (efferent limb) -Loss of taste in the anterior two-thirds of the tongue -hyperacusis (no innervation to the stapedius) CNVIII: exits the brainstem from the pontomedullary junction, lateral to CNVII and enters the cerebellapontine angle. Lesion would cause: -hearing loss and tinnitus from impairment of the cochlear portion of the nerve. -vertgio, dysequilibirum and nystagmus CN IX: exits the brain stem from the postolivary sulcus in the medulla just rostral to the vagus nerve. Injury would cause: -loss of base/sensation in the posterior 1/2 of the tongue. -no gag reflex (afferent limb) CN X: exits brain stem from the postolivary sulcus just caudal to CN IX. Damage would cause: -dysphonia (lack of innervation to the laryngeal muscles) -impaired swallowing with inability to elevate soft palate (lack of innervation to the pharyngeal muscles) -Lack of gag reflex (efferent limb) CN XI: exits the from lateral medulla and upper segment of the cervical spinal cord. Injury would lead to -weakness on head rotation away from the side of the lesion (SCM) -ipsilateral shoulder droop (TRAP) CN XII: exits the medulla from the postolivary sulcus medial to CN IX and X, but caudal to CN IX and rostral to CN X.

11609 Know what nerves comes out of brainstem

MDD -Over 2 weeks -Over 5 of the 9 SIGECAPS -significant functional impairment -no lifetime history of mania -not d/t drugs or medical condition Adjustment disorder with depressed mood -identifiable stressor -onset within 3 months of stressor -marked distress -significant function impairment -doesn't meet criteria for another DSM 5 disorder. Normal stress response -not excessive or out of proportion to severity stressor -no significant functional impairment

11619 What are the differential diagnosis of depressed mood? Hint there are 3 of them.

Latissimus dorsi is innervated by thoracodorsal nerve (C6-8) -Originates in a broad area spanning from the iliac creast and lumbar fascia -Inserts at the bicipital groove of the humerus -Function: extension, adduction and internal rotation of the humorous. a. deltoid is innervated by axillary nerve. It abduct the arm. Injured from sudden or forceful loading of the arm while in abduction b. infraspnatus is innervated by supra scapular. It externally rotate the arm. Common injury d/t to falls or overuse d. trapz elevates, rotates and stabilize the scapula. Innervated by CN XI. Frequently injured in rear-rend "whiplash" motor vehicle accidents e. triceps is innervated by the radial nerve (C6-8, T1) and is the primary muscle involved in extension of the elbow.

11679 Dude got stabbed in the back. The injured muscle assists in internal rotation of the shoulder and is innervated by the thoracodorsal nerve. What muscle is this? a. deltoid b. infraspinatus c. latissimus doors d. trapezius e. tricep

Social anxiety disorder (social phobia) - Anxiety restricted to social and performance situations, FEAR OF SCRUTINY & EMBARRASSMENT Panic disorder - recurrent, UNEXPECTED panic attacks Specific phobia - excessive anxiety about a SPECIFIC OBJECT or situation (phobic stimulus) Generalized anxiety disorder - CHRONIC MULTIPLE WORRIES, anxiety, tension

11751 Key differential diagnosis for Panic disorderS?

Epistaxis are classified as anterior or posterior. -Anterior far more common, vast majority being in the watershed area of the nasal septum (anteroinferior part of the nasal septal mucosa known as the Kiesselbach plexus. Anastomosis of the following vessels: 1. septal branch of the anterior ethmoidal artery 2. lateral nasal branch of the sphenopalatine artery 3. septalbranch of the superior labial artery (branch of the facial artery) Management via compression of the nasal alas. a/b: lateral nasal wall contains the superior, middle and inferior turbinates (aka conchae). These 3 bony projections are covered with mucous membrane that warm, humidify and filter inspired air and expand and contract in response to environmental changes (eg, temp, humidity, allergens). The turbinates form corresponding meatus that serve as drainage pathways. -superior meatus - drains the sphenoidal and posterior ethmoidal sinuses. -middle meatus - drains the frontal, maxillary and anterior ethmoidal sinuses (most common site of nasal polyps) -inferior meatus drains the nasolacrimal duct d/e: branches of sphenopalatine artery supply the posterolateral wall and posterior choanae. These posterolateral branches are most likely source of posterior nose bleed. Posterior epistaxis is usually severe and cannot be treated with cautery.

11783 A 6 yo kid has persistent nasal bleeding. He picks his nose frequently. Examination shows continuous blood trickle from his right nostril. Silver nitrate cautery is performed and the bleeding stops. Cautery was most likely applied to which of the following locations in this patient's nasal cavity? a. inferior turbinate b. middle meatus c. nasal septum d. posterior choanae e. posterolateral wall

Rett is a neurodevelopmental disorder mainly in girls that is characterized by: 1. normal until 5-18 months 2. loss of motor and language skills 3. stereotypic hand movement 4. Deceleration of head growth a. autism spectrum disorder is characterized by impaired social communication/interactions and restricted, repetitive interests or behaviors typically presenting by age 3. Autism doesn't explain this patient decelerated head growth, stereotypic hand movement and regression from earlier milestones b. fragile X: CGG triple repeat. Features include intellectual disability, a long narrow face, large protruding ears, macrocephaly and macroorchidism. Leech-Nyhan is X-linked recessive def in HGPRT, leading to increase uric acid. Clinical features are gout, poor muscle control, intellectual disability, writhing or repetitive involuntary movements, and self-mutilaitng behaviors.

11796 A child is globally hypotonic and unable to sit upright without being held. She doesn't make eye contact with the physician and makes no sound apart from occasional grunt. She wrings her hands repeatedly. Her height and weight are in the 60th percentile. Head growth is at 40th percentile. Which of the following best describes what this patient has? a. autism spectrum disorder b. Fragile X c. intellectual disability d. Lesch Nyhan e. Rett f. stereotypic movement disorder

Recent history of viral esophagitis and pneumocystis pneumonia = HIV infection. -Headache, confusion and inflammatory CSF = meningitis, most likely cause by CRYPTOCOCCUS NEOFORMANS, an opportunistic fungus that causes meningitis (or meningoencephalitis) in immunosuppressed patients. Morphology: YEAST form ONLY! Round/oval encapsulated cells with narrow based buds. Virulence factor: thick polysaccharide caps Epidemiology: present in soil and pigeon droppings. Opportunistic and respiratory transmission Infection: Lung is primary site. Most common is meningoencephalitis if disseminated Diagnosis: 1) India ink stain 2) latex agglutination 3) Sabouraud's agar 4) Methenamine (GMS), mucicarmine stains of tissue Treatment: 1) Amphotericin B and flu cytosine (acute meningitis) 2) Fluconazole for lifelong prophylaxis. a. nonseptate that branch at wide angels = Nucor and Rhizomes species. These fungi cause infection of the paranasal sinuses in immunosuppressed patients b. Candida albicans forms germ tubes (sprouts of true hyphae from east cells) if incubated in 37 degrees. This test helps to different C. albicans from other Candida species. d. Spherules (round structures containing endospores) are found in the tissue form of Coccidioides immitis, which causes lung infection and disseminated mycosis, but it is not associated with meningitis. e. Sporangium is a structure that produces and contains spores. It is present in mold fungi. Cryptococcus has only a s yeast form and thus doesn't have sporangia.

118 A 54 y.o male has one week history of headaches and progressive confusion. Dude has viral esophagitis six months ago and pneumocystis pneumonia two months ago. Lumbar puncture is performed that shows increase CSF protein concentration and CSF pleocytosis. The latex agglutination test is positive for soluble polysaccharide antigen. LM would most likely reveal what? a. nonseptate hyphae b. germ tubes c. budding yeast d. spherules e. sporangium

The pathway is form th internal carotid artery, ophthalmic artery and then the retinal artery. Retinal artery occlusion (RAO) is an important cause of acute, painless, monocular vision loss. Thromboembolic complications of atherosclerosis in the internal carotid are the most common cause of RAO. The retinal artery is one of the first branches of ophthalmic artery, which receives its supply from internal carotid. The retinal artery then travels within the retinal nerve to supply the inner retina and the surface of the optic nerve.

11832 A 71 y.o man suddenly lost vision on his left eye for one hour. PMH coronary artery disease, HTN and type 2 DM. Blood pressure is 145/80 mm Hg. ON examination, the patient can only see hand motions through the left eye. Funduscopic shows cherry red spot in macula with surrounding retinal whitening. Cranial nerve examination is otherwise unremarkable. There is a left-sided neck bruit on CVS examination. Which of the following is the most lily path of the embolus causing this patient's symptoms?

This patients has cardigenic shock d/t acute right ventricular MI (RVMI). -This is seen with 30-50% of patients with acute inferior wall MI d/t occlusion of the proximal right coronary artery before the origin of the RV branches. patients typically present with hypotension, distended jugular veins and clear lungs d/t RV dysfunction. Ischemia or infarction of the RV results in decrease RV stroke volume, which in turn leads to diminished LV filling and CO in spite of normal LV function. RV dilation and elevated diastolic pressures also cause a shift of inter ventricular septum toward the LV cavity, exacerbating LV filling and CO causing systemic hypotension and shock. Patients will have elevated central venous pressure and a normal or decreased pulmonary capillary wedge pressure (PCWP). b/d. decreased CO with elevated PCWP is typical seen in acute MI with significant LV systolic dysfunction. The elevation in pulmonary venous pressure often cause cardigenic pulmonary edema (crackles on asuculatiaotns)

11833 A 66 y.o man has chest pain and nausea. BP is 70/60 and pusle is 60. Lungs are clear on auscultations. ECG reveals ST segment elevation in leads II, III and aVF. Chest x-ray is unremarkable. He is diagnosed with an inferior wall MI. Emergent cardiac cauterization shows complete occlusion in the proximal right coronary artery. The patient is persistently hypotensive in the cardiac cauterization lab despite rapid infusion of 500 mL of normal saline. Which of the following hemodynamic changes is most likely to be observed in this patient? a. decreased CO & PCWP and increase central venous pressure b. decrease CO. PCWP and central venous pressure increase. c. increased CO and PCWP. Decreased central venous pressure d. decreased CO and central venous pressure. Increased PCWP e. increased CO. Decreased PCWP and central venous pressure.

Mitochondrial encephalopmyopathy: neuromuscular lesions, ragged skeletal muscle fibers and lactic acidosis. Mitochondrial disorders follow a maternal inheritance pattern because an embryo's mitochondria are inherited from the ovum only. Mitonchdronia contain their own genome, which is vulnerable to mutations. Each cell has hundred of mitochondria, and defects in their genome may occur in any number of the mitochondria within the cel. HETEROPLASMY: describes the condition of having different mitochondrial genomes within a single cell. The severity of mitochondrial diseases is often directly related to the proportion of abnormal to normal mitochondria within a patient's cells. a. anticipation refers to increased severity or earlier onset of an inherited disease in successive generations. c. penetrance is the probability that a person with a given genotype will express its associated phenotype. If all individuals within gene express its phenotype , that gene is said to have complete penetrance. d. Mosaicism refers to the presence of 2 or more cell lines, each with unique nuclear genome, within the same individual. While patients with both somatic and gremlin masocicsm can demonstrate disease traits and also pass the disease on to their offspring, it doesn't best explain the pattern of female only transmission. e. uniparental disomy occurs when both members of a chromosomal pair are inherited from one parent, which can cause problems d/t genomic imprinting. For instance, although most often d/t chromosomal deletions, uniparental disomy can also cause Prader-Willi and Angolan syndrome d/t loss of expression of maternal/paternal imprinted components of a critical region of chromosome 15.

1935 A 26 y.o woman is being evaluated for possible inherited disorder. She has a 6 year history of generalized tonic clonic seizures, and a year ago, she had partial loss of vision d/t an occipital infraction. Her motor has chronic intermittent muscle weakness and lactic acidosis, and her maternal uncle has hemiplegia. Skeletal muscle biopsy of the patients shows ragged appearing muscle fibers. After further evaluation, all the affected family members are found to suffer from the same inherited disease. Which of the following is most likely explanation for the variability of clinical manifestations in the affected family members. a.. anticipation b. heteroplasty c. incomplete penetrance d. mosaicism e. uniparental disomy

Obstructive sleep apnea (OSA) is characterized by recurrent episodes of UPPER AIRWAY COLLAPSE during sleep. Etiology: Anatomical and neuromuscular abnormalities. Path: -Neuromuscular weakness as a pathogenic mechanism in OSA is supported by the fact that apneas occur only during sleep, a time of muscle relaxation. The upper airway dilator muscles weaken during the transition from wake to sleep, leading to airway narrowing and ultimately collapse in individuals with OSA. Stimulation of the hypoglossal nerve using an implantable nerve stimulator causes the tongue to move forward slightly, increasing the anteroposteiro diameter of the airway. Studies with these devices have shown a reduction int he number of obstructive events during sleep. b/c: the lingual nerve arises from the mandibular division of the trigemnial nerve and supplies sensory innervation to the tongue. The maxillary division of the trigeminal nerve provides sensation to the mid-face area. Neither of these nerves provides motor innervation to the oropharyngeal muscles involved in OSA. d. the phrenic nerve supply the diaphragm. Although diaphragmatic paralysis can cause sleep apnea, OSA with loud snoring and gasping respirations is d/t oropharyngeal rather than diaphragmatic dysfunction. e. the vocal cords are innervated by the recurrent laryngeal nerve, but sleep apnea is primarily a problem of the pharynx, not larynx.

11845 A 57 y.o man has been complaining of unrefreshing sleep. His wife says he snores loudly during sleep and frequently gasps for breath. He also has severe claustrophobia. PMH HTN. Pliant takes no sedative meds and is a lifetime nonsmoker. Blood pressure is 156/94 and BMI is 30 kg/m2. Physical examination is significant for a bulky tongue and crowded, narrow oropharynx. Electrical stimulation of which of the following nerves may improve the pathophysiologic cause of this patient's symptoms? a. hypoglossal b. lingual c. maxillary d. phrenic e. recurrent laryngeal

The inpatient physician's failure to note and address the abnormality is a preventable medical error that resulted in delayed diagnosis. Preventable medical errors involve harm to the patient by an act of commission or mission rather than from the underlying disease and are the result of failure to follow evidence based best practice guidelines. a. malpractice is a legal determination; it involves treatment provided at below the accepted standard of practice and has resulted in injury or death to the patient. It is not a category of medical error but rather refers to the consequence of many different types of errors that result in harm. b. a near miss is a medical error that is caught by someone before any harm is done (eg, patient is prescribed a lethal dose of med, but the error is caught by the pharmacist). c. a non-preventable adverse event is a complication that cannot be prevented given the current sate of medical knowledge (eg, an allergic reaction to a med in a patient with that has NKA) e. sentinel event is an unexpected occurrence involving death or serious physical or psychological injury (eg, inpatient suicide, death of a full term infant, retained object after surgery) that required immediate investigation.

11847 A 40 y.o woman has hypothyroidism but her pathology was overlooked by her physician who didn't check her TSH level. Which of of the following is appropriate categorization for this type of medical error? a. malpractice b. near miss c. non-preventable adverse event d. preventable adverse event e. sentinel event

TP: people who has the disease and obtained a positive test. FN: people who has the disease and obtained a negative test (meaning the test failed to identify that they are sick) PPV (positive predictive value): is the test's ability to correctly identify those with the disease from all those who had positive results. It is the fraction of those truly with the disease among those with positive test. It is dependent on the prevalence of the disease. Sensitivity: rules out those with a disease. Very important for screening purposes. A test with high sensitivity is one which identifies most patients with the disease. Therefore, a high sensitivity test will positively identify most of the sick people. It is not dependent on prevalence Specificity -excludes those without a disease. A very specific test is one which has a low false positive rate. In a high specificity test most healthy patients will have a negative test result. Specificity is not depend on prevalence. If you lower cutoff point of r serum biomarker level, then you are going to have more positive results, thus increasing the sensitivity of the test. However, false positive will have a relatively larger increase, thus PPV and FN will also decrease.

1191 What are true positive, false negative, PPV, sensitivity and specificity?

NF-kB is a family of transcription factors that perform critical role in the immune response to infection and inflammation. In inflammatory cells, NF-kB is normally present in a later, inactive state bound to its inhibitor, IkB. When extracellular signal, such as binding of bacterial antigens to a toll like receptor, causes activation of IkB kinase. This results in ubiquitination and subsequent destruction of IkB with release of free NF-kB. Once free, NF-kB enters the nucleus and promotes the synthesis of number of inflammatory proteins such as cytokines, acute phase reactions, cell adhesion molecules, and leukocyte related growth factors. The inflammatory cascade is self-limiting as NF-kB also stimulates the transcription of more IkB, ultimately rebinding the freed NF-kB. a/e: G-CSF is the principal protein the stimulates the production and relate of neutrophils from bone marrow. TNF-alpha plays a role in response to infection by increasing neutrophil chemotaxis and stimulating macrophage phagocytosis. Although nfkB up regulates transcription of G-csf AND tnf-A, IKB DEGRADATION IS NOT DIRECTLY INVOLVED IN ACTIVATION OF THESE FACTORS B. JAK2 is a TK involved in the signaling pathway for myeloproliferation. Constituent activation of JAK 2 is associated with polycythemia vera, essential thrombocytosis and myelofibrosis. It doesn't play a major role in the immune response to infection d. although transforming growth factor -beta (TGF-beta) plays a role in immune response and is triggered by inflammatory pathways, it is complexed to TGF-beta binding protein and latency associated peptide not IkB.

11955 A 61 y.o man has fever, chills and productive cough with thick blood tinged sputum for the past several days. Culture revealed dude has Klebsiella pneumoniae. It is determined that the bacteria express a lipopolysaccharide on their outer membrane surface that stimulates tool like receptor in the inflammatory cells. This in turns leads to the degradation of the IkB inhibitor, which normally binds to a latent transcription factor found in the cytoplasm. Which of the following factors is most likely to be directly activated by the removal of this inhibitor protein? a. granulocyte colony stimulating factor b. Janus Kinase 2 c. NFkB d. Transforming growth factor-beta e. TNF-a

Sickling is promoted by conditions associated with 1. LOW OXYGEN levels 2. increased acidity 3. low blood volume (dehydration) Sickled cells are not flexible enough to pass through small blood vessels. Organs where blood moves slowly (eg, spleen, liver) are predisposed to lower oxygen levels or acidity. Organs with high metabolic demands (eg, brain, muscles, placenta) promote sickling by extracting more oxygen from the blood (OXYGEN UNLOADING). a/c: 2,3-BPG binds to 2 beta global chains and stabilizes the taut (T) deoxyhemoglobin, this biding decreases hemoglobin's oxygen affinity, facilitating oxygen relate at the tissue level. Depletion would cause an increase in hemoglobin affinity to oxygen (left shift), which would result in uptake of oxygen by hemoglobin. Therefore, sickling will decrease. -Similarly, increased acidity or low pH is associated with sickling, so decreased acidity with elevated pH would not promote sickling.

1241 In which of the following situation would a sickle cell patient be more likely to have a sickle cell crisis? a. 2,3-bisphosphoglycerate depletion b. beta globin chain folding c. capillary pH values >7.4 d. interaction with fetal hemoglobin e. oxygen unloading

Enteropeptidase, a jejunal brush border enzyme, is repsonbile for activation of Trypsin, from trypsinogen. Once active, trypsin can cleave peptide bonds in dietary proteins and activate the other pancreatic enzymes. Enteropeptidase def leads to both protein and fat malabsorption as trypsin is required to activate enzyme required for both lipid and protein digestion. Symptoms are diarrhea, failure to thrive and edema. a. amylase is made in salivary glands and pancreas; it catalyzes the hydrolysis of complex carb to simple sugar b. bile salts are formed through bile acid conjugation with amino acids glycine and taurine. This increases their solubility, allowing them to aid in lipid digestion and absorption through the formation of micelles. c. lactase is produced by the intestinal brush border and is responsible for hydrolysis of lactose to glucose and galactose. d. chief cells makes pepsinogen which is cleaved by Hal in stomach into pepsin that initiate protein digestion. e. secretin is a peptide hormone secreted by S cells of the duodenum; it stimulates the secretion of HCO3 from the pancreas and inhibits gastric acid secretion in the stomach. Neutralization of the acidic pH of food entering the duodenum from the stomach is necessary for the proper function of pancreatic enzymes.

1251 A 3 m.o boy has large volume diarrhea and failure to thrive. The patient was born via spontaneous vaginal delivery at 38 weeks gestation. PE shows mild abdominal distention, muscle wasting, and pitting edema. Histochemical evaluation reveals absence of enteropeptidase activity in the proximal intestinal villi. Activity of which of the following substances is most lily impaired in this patient? a. amylase b. bile salts c. lactose d. pepsin e. secretin f. trypsin

This patient has typical symptoms of hypoglycemia (eg, disorientation, sweating, tremors, palpitations), which are relieved by intake of glucose. the 3 most important predisposing factors for hypoglycemia in patients with type 1 dm are excessive insulin dose, inadequate food intake, and physical activity/exercise. GLUT-4 expressed on skeletal muscle cells are translocated to cell membrane and transverse tubules (deep invaginations in the cell membrane) in response to insulin. It translocation also occurs during muscle contraction by insulin independent mechanism, which is mediated by several cellular factors, including AMP-activated kinase, nitric oxide, and calcium calmodulin activated protein kinase. In normal individuals, over hypoglycemia doesn't occur with exercise because a drop in blood glucose will stop insulin release from the beta cells and counter-regulatory hormones (eg, glucagon) will increase endogenous glucose production via glycogenolsyis and gluconeogenesis. However, patients taking exogenous insulin are vulnerable to exercise induced hypoglycemia as insulin will continue to be released from the injection site despite falling glucose levels. In addition, strenuous exercise may cause changes in skin perfusion that can lead to increased insulin absorption (especially if the insulin is injected into an exercising limb rather than the abdominal area). b/c/d. infection, pain and sleep deprivation tend to cause hyper-rather than hypoglycemia. Stressful situations increase catecholamine release, which raises glucose by decreasing pancreatic insulin secretion and by increasing glycogenolysis and gluconeogeneiss. e. weight gain is associated with insulin resistance insulin-treated patients who gain weight usually develop hyperglycemia and require increased doses of insulin to maintain control of glucose

1325 A 27 y.o male has type 1 dm. He has been on a stable insulin regimen for the last 4 years. Since his last visit, the patient has experienced frequency episodes of disorientation, palpitations, tremulousness, and excessive sweating, all of which resolve quickly after drinking some honey dissolved in water. Which of the following factor most likely precipitated this patient's symptoms? a. exercise b. moderate to severe pain c. respiratory infection d. sleep deprivation e. weight gain

Methionine can be metabolized into S-adenosyl-methionine (SAM), which acts as a methyl donor for many methyltransferase reactions. After transfer of methyl group, SAM is converted into S-adenosyl-homocysteine (SAH), which is broken down to form adenosine and homocysteine. Homocysteine can be converted to cystathionine (via cystathionine synthase and B6). Cystathionine can then be converted into cysteine (via cystathionase and B6). Homocysteine can be converted back into methionine (via methionine synthase and B12). Homocystinuria is a condition that can lead to hyper coagulability and thromboembolic occlusion b/c homocysteine is prothrombotic. People with deficiency of methionine synthase can develop premature acute coronary syndrome, ectopic lentil (ocular lens displacement) and intellectual disability. -this is the most common cause of homocystinuria, thus cysteine is an essential amino acid to them. -hypermethioninemia a. asparagine synthase converts aspartate to asparagine, the amino acid that is essential for rapidly dividing tumor cells that cannot produce it quickly enough on their own. The chemotherapy drug asparginase decreases asparagine contraption in tumor cells and leads to lysis of these rapidly growing cells. c,d,f. maple syrup urine (def of alpha-ketoacid DH). Def can lead to buildup of branched chain amino acids (valine, leucine and isoleucine) and their metabolites, resulting in feeding difficulties, seizures, cerebral edema and sweet odor urine e. phenylalanine hydroxylase catalyzes the hydroxylation of essential acid phenylalanine into tyrosine. Def of phenylalanine hydroxylase is the most common cause of phenylketonuria, which results in severe intellectual disability if left untreated.

1332 A 12 y.o boy has severe chest pain with history of intermittent substernal chest pain for the past few months that typically occurs after heavy activity. ECG shows ST segment elevations in leads II, III, and aVF. After acute stabilization and treatment, further labs workup shows an increased serum of methionine level. Which of the following amino acids is most likely essential in this patient? a. asparagine b. cysteine c. isoleucine d. leucine e. tyrosine f. valine

A major determinant of viral tissue tropism is its surface proteins (on the viral envelope) ability to bind to complementary host cell receptors. Mutation in viral encoded evolve glycoprotein can therefore dramatically affect the range of host cells that the virus can attach to or infect. Ex: hemagglutinin of influenza A strain that was previously confined to domestic livestock. If the mutation conferred a new binding affinity for a neuraminic acid contacting glycoprotein on the surface of human nasopharyngeal epithelium, then human would be vulnerable to infection. b. with developed viruses, the nucleocapdis protein surrounding the viral genome is not directly involved in viral attachment. d. Enveloped viruses with ss(-)RNA has different methods to initiate genomic transcription. -some have a viral encoded endonuclease associated with the viral RNA-dependent RNA polymerase. This endonuclease is thought to cleave short 5' RNA fragments that are used to prime the viral RNA for transcription by the viral RNA polymerase. -Host range of infectivity of an orthomyxovirus could conceivably be altered by an endonuclease mutation. However, other enveloped RNA viruses - including this eiwth negative and those with positive sense can synthesize new viral genomic and mRNA without endonuclease involvement. Thus, the host range of infectivity demonstrated by the enveloped RNA viruses is not likely to be affected by a viral endonuclease mutation

1411 After exposure to mutagens, several strains of enveloped RNA viruses isolated from the nasal exudate of experimental animals acquire the ability to infect human epithelial cells. Which of the following viral-encoded proteins is most likely to have mutated in this circumstance? a. surface glycoprotein b. capsid protein c. RNA polymerase d. endonuclease e. protease

Tay-Sachs is an AR neurodegenerative disorder commonly seen in the Ashkenazi Jews. beta-hexoaminidase A def, resulting in accumulation of membrane glycolipid GM2 ganglioside within lysosome. Patients are usually asymptomatic in the first few months, followed by progressive neurologic deterioration as glycolipids accumulate in the brain. Clinical consequences: -weakness, hypotonia, developmental regression, seizures, blindness and spasticity. -macrocephaly and abnormal startle reflex -cherry red macula where fovea is surrounded by white macula appearing as a halo. The halo is d/t ganglioside buildup in ganglion cells. a. galactocerebroside builds up in Krabbe disease. Neurodegenration like Tay Sachs however, patients will also have peripheral neuropathy and optic atrophy. b. globotriaosylceramide accumulates if Fabry. Angiokeratomas (benign cutaneous lesion of capillaries), peripheral neuropathy and glomerulopathy and typically present in adult hood. c/g: cherry red macula is also found in Gaucher and Riemann-Pick. These guys however have hepatosplenomegaly -beta-glucocerebroside accumulates in Gaucher -Sphingomyelin accumulates in Neimenn-Pick d. glycogen storage diseases include von Gierke disease and Pompe. -von Gierke (glucose-6-phsphatase def) typically have hepatomegaly, hypoglycemia, seizures and/or lactic acidosis. -Pompe disease (lysosomal alpha-1,4-glucosidase def) classically cause cardiomegaly and severe hypotonia. f. mucopolysaccharidosis (eg, Hurler syndrome, Hunter syndrome) are lysosomal storage diseases that cause build up of GAGs such as heparin and derma tan sulfate. GAG accumulation causes nerucognitive decline as well as coarse facial features and skeletal abnormalities.

8524 An 8 m.o girl has irritability and regression of motor skills. She started easily with loud noises. She has macrocephaly. Funduscopic shows bright red fovea centralism that is surrounded by contrasting white macule. Peripheral vision is decreased. Accumulation of which of the following metabolites is most likely present in this patient's tissues? a. galactocerebroside b. globotriaosylceramide c. glucocerebroside d. glycogen e. GM2 ganglioside f. heparin sulfate g. sphingomyelin

Cyanide toxicity is dependent upon its ability to bind to Fe+3 (ferric) with high affinity, inhibiting cytochomre c oxidase in the mitochondria. This inhibits oxidative phosphorylation causing lactic acidosis and death as a result of cells switching to anaerobic metabolism Clinical presentation -REDDISH SKIN DISCOLORATION -tachypnea -headache -tachycardia -nausea/vomiting, confusion and weakness. Labs: lactic acidosis in conjunction with narrowing of the venous arterial PO2 gradient, resulting for the inability of tissue to extract arterial oxygen. Treatment: inhaled amyl nitrite oxides ferrous iron present in hemoglobin to ferric iron generating methemoglobin. Methemoglobin is incapable of carrying oxygen but has high affinity for cyanide; it binds and sequesters cyanide in the blood, freeing cytochrome oxidase and limiting its toxic effects. Hydroxycobalamin, a vitamin B12 precursor, and sodium thiosulfate are also antidotes for cyanide poisoning. there interaction with cyanide generate relatively nontoxic metabolites that are easily excreted in the uric. a/b: 2,3-BPG causes a rich shift in the oxygen-hemoglobin dissociation curve, decreasing hemoglobin affinity to O2 but increasing its affinity to CO@ c. Hemoglobin has much higher affinity for CO than it does for oxygen. This is the basis for carbon monoxide poisoning, which is treated with high for oxygen. e. the affinity of hemoglobin for iron is not affected by nitrite administration, although nitrites do oxide the heme iron to its ferric state. f. Lead poisoning causes defect heme synthesis. If lead is acutely ingestion, chelation therapy with dimercaprol and edentate disodium calcium (CaNaEDTA) should be initiated. 0

1415 A 34 y.o industry worker is experiencing rapid onset of sob, dizziness, palpitations and flushed skin after accidental exposure to chemical fumes. PE tachypneic and has reddish discoloration of the skin without cyanosis. Patient is immediately given inhaled nitrate from a safety kit. The success of this antidote is dependent upon its ability to convert hemoglobin into a form with increased affinity for which of the following substance. a. 2,3-biphosphoglycerate b. CO2 c. carbon monoxide d. Cyanide e. iron f. lead

Recombination - exchange of genes between 2 chromosomes via crossing over within homologous regions. This only occurs in nofsegmented viruses. c. Reassortment (segmented viruses) -is where they exchange whole segments of chromosome (eg, the flu). a. interference is when one virus inhibits a second virus from replication and/or release that is infecting the cell.. b. phenotypic mixing can occur when a cell is infected by 2 viral strains and progeny visions containing parental genome from one strain and nucleocapsid (or envelope) proteins from the other strain. This may result in progeny acquiring additional traits (eg, enhanced ability infect new shot ells d/t different nucleocapsid protein). However, as the genome is unchanged, subsequent progeny would no retain these traits. e. transformation is the uptake of naked DNA by a prokaryotic or eukaryotic cel. In virology, transformation also describes incorporation of viral DNA into a host cell chromosome. This alters the genetic composition of the shot cell but typically causes no genomic change in progeny virions.

1461 Virologists are studying acyclovir resistant HSV. In an experiment, resistant HSV type 2 strain with a defective thymidine kinase gene is mixed in with a nonresistant HSV type 1 strain They found that some type 1 virions acquire resistant to acyclovir. What mechanism best explains this finding? a. interference b. phenotypic mixing c. reassortment d. recombination e. transformation

This patient with intermittent nocturnal dyspnea, episodic cough and FH of atopy may have asthma despite his normal spirometry findings. Asthma is an obstructive airway disease that occurs d/t hypersensitivity of the airways to various stimuli, including physical, chemical and allergenic irritants. Personal or FH of other disease in allergic triad (allergic rhinitis, atopic dermatitis, allergic asthma) can aid in raising the suspicion for asthma. The diagnosis of asthma is typically confirmed by spirometry. Patients with asthma will demonstrate decreased FEV1 and peak expiratory flow rate on spirometry. These changes typically reversible with the use of a bronchodilator, usually an inhaled beta-adrenergic agonist such as albuterol. When patient present with a history consistent with asthma but has normal spirometry values, bronchoprovocation techniques such as methacholine or histamine administration ,exercise or cold air inflation can considered to elicit asthma symptoms and confirm th diagnosis. Methacholine is a msucarininc cholinergic agonist that causes bronchoconstriciton and increased airway secretions; a decrease in FEV1 after methacholine challenge indicates bronchial asthma. a/e: ipratropium bromide and scopolamine are antagonist of muscarinic cholinergic receptors. Ipratropium is typically used in an inhaled form to treat bronchoconstriciton in patients with obstructive lung disease such as asthma or emphysema, and scopolamine is used for motion sickness b. lavalbuterol is a beta-2 adrenergic agonist used in treatment of asthma and is sometimes sued in patients with adverse reactions to albuterol d. phenoxybenzamine is a nonselective alpha-adrenergic antagonist used in treatment of pheochromocytoma.

1523 A 43 yo man has nighttime dyspnea that awakens him. This started about a year ago after a severe upper respiratory tract infection, and he now has noticed a new episodic cough. PMH is unremarkable for other med problems and allergies, but his older brother has atopic dermatitis. Pulmonary function test are as follows: Forced expiratory volume in 1 second (FEV1) is normal. FEV1/Forced vial capacity ratio is normal. Diffusion capacity for carbon monoxide is normal Administration of which of the following medications would be most helpful during the reminder of this patient's diagnostic workup a. ipratopium bromide b. levalbuterol c. methacholine d. phenoxybenzamine e. scopolamine

Fever, pharyngitis and lymphadenopathy in young adult = infectious mononucleosis. Morphology: atypical lymphocytes on the peripheral smear. Abundant pale blue cytoplasm with a basophilic rim that is often indented that is often indented by surrounding RBCs. EBV is associated with both Hodkin's and non-Hodgkin's lymphomas as well as nasopharyngeal carcinoma. a. anal carcinoma = HPV 16, 18 and 31 b. glomerulonephritis = strep c. myocarditis and pharyngitis = C. diph e. retinas = CMV in immunocompromised (especially HIV positive) patients. f. tenosynovitis = N. gonorrhoeae

1572 Patient has infectious mononucleosis. What is an associated disorder associated with this infection? a. anal carcinoma b. glomerulonephritis c. myocarditis d. nasopharyngeal carcinoma e. retinitis f. tenosynovitis

Dude has HIV given his oral thrush (usually seen with CD 4 count lower than 200), cervical and inguinal lymphadenopathy, and brain lesions. HIV patient with headaches, seizures, and multiple ring enhancing CNS lesion on MRI suggest tosoplasmic encephalitis. Seizures ar common complication of such brain lesions and their surrounding edema. Extensive pet (cat) exposure is not essential for the diagnosis, as toxoplasmosis can also be transmitted through contaminated foods. a. aspergillum is not a leading opportunistic pathogen in patients with HIV, possibly because they have relatively intact phagocytic function despite their T cell dysfunction. Aspergillum typically affects the lungs; however, ti can cause brain abscesses in patients who are immunocompromised d/t neutropenia or transplant. b. Glioblastomas are ring enhancing, they are typically solitary and often have characteristic butterfly appearance. c/d: neurocysticercosis (d/t Tania sodium (pork tapeworm) ingestion) can cause multiple brain lesions but would be unlikely in this patient with no travel history to South and Central America. Similarly, TB cis infection, which can cause similar lesion, si a consideration in developing countries but is a less lily diagnosis in the US

1573 A 42 y.o man has generalized tonic-clonic seizure. He has no prior history of seizure but has had headaches for the past several weeks. He smokes a pack of cigs a day and drink 3-4 cans of beer every even gin. PE is notable for a postictal state, oral thus and several enlarged cervical na inguinal lymph nodes. Brain shows multiple ring enhancing lesions. What does this patient has? a. Aspergillus b. glioblastoma multiform c. TB d. neurocysticercosis e. toxoplasmosis

PAO2 (alveoli) = 104 mm Hg pO2 generally reaches PAO2 by the time it passes through the first third of the alveolar capillaries d/t a high rate O2 diffusion through the respiratory membrane. However, by the time oxygenated blood enters the systemic circulation, the pO2 drops to about 100 mm Hg d/t addition of deoxygenated blood from the bronchial circulation, intrapulmonary arteriovenous anastomoses, and Thebesian veins of the heart. The discrepancy between alveolar and arterial O2 concentration is termed (A-a) gradient, which is normally between 5-15 mm Hg, with older individuals having a higher A-a d/t age related decrease in O2 diffusing capacity. This patient has low PaO2 and PAO2 and a normal A-a gradient (71-60=11). This indicates that his low PaO2 is directly d/t his low PAO2 and not caused by V/Q mismatch or O2 diffusion impairment. Possible causes of hypoxemia in the setting of normal A-a gradient include 1. alveolar hypoventilation common in patients with suppressed central respiratory drive (eg, sedative, overdose, sleep apnea) or those with diseases that decrease inspiratory capacity (eg, myasthenia gravid, obesity). 2. inspiration of air at high altitude a. diffusion impairment can be cause by thickening of the alveolar capillary membranes, such as alveolar hyaline membrane diseases. A-a gradient will increase because O2 in the alveoli cannot be transported into blood. c. left to right shunt happens when the left heart (or systemic arterial circulation) is shunted into the right heart (or pulmonary artery). because blood in the left heart is oxygenated, left to right shunts doesn't lead to hypoxemia. This can occur with atrial or ventricular septal defects or patent ductus arterioles d. right to left shunt occurs when venous blood bypasses the lung and enters the arterial circulation, thereby decreasing PaO2. The A-a gradient would increased. This type of hypoxemia occurs in patients with cyanotic congenital heart defects e. V/Q mismatch is a common cause of hypoxemia. Poor ventilation of a lung segment with normal perfusion (eg, pneumonia, COPD) results in physiologic shunting, leading to increased A-a gradient.

1582 A 52 y.o homeless man was found unresponsive in the street. BP is 108/62. ABG partial pressure of oxygen in his arterial is 60. The partial pressure of oxygen in his alveoli is 71. Which of the following is the most likely cause of this patient's symptoms? a. diffusion impairment b. hypoventilation c. left to right shunt d. right to left shunt e. ventilation/perfusion mismatch

Statins serious side effects are: 1. myopathy- muscle pain with increase creatine kinase 2. hepatitis With exception to pravastatin, the statin drugs (simvastatin, lovastatin, atorvastatain) are metabolized by P-450 3A4. Erythromycin inhibits cytochrome 3A4, thus concurrent use will increase statin levels, which increase the risk of developing myopathy. Rhabdomyolysis can cause acute renal failure. all the other ones are CYP450 inducers.

161 A 57 y.o Caucasian male is hospitalized with muscle pain, fatigue and dark urine. PMH stable angina. Meds include metoprolol, atorvastatin and aspirin. Lab shows acute renal failure The addition of which of the following meds is most likely to have precipitated this patient condition? a. erythromycin b. phenytoin c. rifampin d. griseofulvin e. phenobarbitone

Lipoatrophy (lipodystrophy) is redistribution of fat from the extremities to the trunk associated with HAART (highly active antiretroviral therapy) and can be considered a product of 2 independent processes: wasting of fat from the face and extremities and deposition of fat in the trunk and viscera. 1. lipoatrophy is primarily seen in patients taking nucleoside RTIs (stavudine and zidovudine) and is also associated with protease inhibitor use 2. central fat deposition also occurs in some patients and causes increased abdominal girth and a buffalo hump (dorsocervical fat accumulation) that is similar to Cushing syndrome. Central fat deposition may be seen with any HIV treatment regimen and may be result of treatingg HIV rather than specific medication adverse effect. Both of these syndromes have been associated with metabolic abnormalities including insulin resistance, hyperTAGs and reduced HDL. b. statins is a HMG CoA reductase inhibitors that are associated with myalgia and an increase in hepatic transaminase levels. c. thiazide (eg, chlorthalidone and hydrochlorothiazide) can cause hyperglycemia and hypeterTAG and may have a small effect on fat distribution. However, these effects have not been seen with loop diuretics (eg, furosemide). d. unlike glucocorticoids, NSAID doesn't cause this shit e. PPI are use to reduce gastric acid secretion in patients with peptic ulcers or GERD, which can cause hypomagnesemia and increased osteoporosis f. thyroperoxidase inhibitor (eg, methimazole and propylthiouracil) are used to treat hyperthyroidism. Common side effects are bone marrow suppression (eg, anemia and agranulocytosis)

1673 A 48 y.o man is having change to her body habits. Her legs are like sticks while her belly is getting bigger. She feels increasingly fatigued but does' have excessive daytime somnolence. She has been homeless for much of the last 5 years. ON PE she is distortable and has an inpprirate affect but is cooperative. There is loss of adipose tissue from the extremities and face, with noticed increase in abdominal girth. What meds is causing this? a. antiretroviral medications b. HMG-CoA redcuatase inhibitor c. Loop diuretic d. NSAIDs e. PPI f. SSRI g. thyroperoxidase inhibitor

The radial nerve enters the forearm anterior to the lateral epicondyle (near the humeroradial articulation) and divides into the: 1. superficial which provides purely somatic sensory innervation to the radial half of the dorsal hand. 2. deep branch that innervates the extensor compartment muscles in the forearm. After passing through the supinator canal (eg, between the superficial and deep parts of the supinator muscle), the deep branch continues to become the posterior interosseous nerve, which innervates muscle involves extension of the finger and thumb. Injury to the radial nerve during its passage through the supinator canal may occur d/t repetitive pronation/supination of the forearm (eg, frequent screwdriver use), direct trauma or dislocation of the radius. Clinical presentation -weakness on finger and thumb extension ("finger drop"_ -tricpes brachia (elbow extension) and extensor carpi radials longs (wrist extension) are typically preserved as the radial nerve branches supplying these muscles come off proximal to the supinator canal. -Cutaneous sensory branches are similarly preserved. A/E: Injury to the radial nerve at the axilla (eg, "crutch palsy") typically causes weakness of the forearm, ahdn and finger extensor muscles (eg, wrist drop, absent trips reflex) with sensory loss over the posterior arm, forearm and dorsolateral hand. Injury to the nerve at the mid shaft humerus (eg, radial groove) usually causes weakness of the hand/finger extensor muscles with sparing the triceps brachia and sesonry loss over the posterior forarm/dorsolateral hand. b. carpal tunnel syndrome can result form any condition that reduces the size of the carpal tunnel and compresses the median nerve (eg, pregnancy, hypothyroidism). patients typically have pain/apresthesias affecting the first 3-1/2 digits. Thenar atrophy with weakness on thumb abduction/opposition may also be seen. c. the coracobrachialis muscle lies deep to the biceps brachia and is perforated and innervated by the musculocutaneous nerve. Nerve injury may result in decreased strength on forearm flexion and sensory loss over the lateral forearm. d. In the wrist, the ulcer nerve passes between the hook of hamate and the pisiform bone in the fibroosseous tunnel known as Guyon's canal. Ulnar nerve injury at this site can cause weakness on finger abduction/adducition and clawing of the 4-5th digits. g. fracture of the surgical neck of the humerus is usually associated with axillary nerve injury. Patients may have weakness of the deltoid and trees minor muscles as well as loss of sensation in the lateral upper arm.

1694 A 35 y.o right handed mechanic comes complains of progressive right hand weakness. Patient uses a screwdriver to perform his duties at work. Neurologic examination, sensation is preserved in his upper limbs. There is weakness on extension of the fingers and thumb in the right hand. Strength is otherwise intact throughout. Triceps reflexes are plus and bilaterally symmetric. What nerve affected in this patient was most likely injured at which of the following locations? a. axilla b. carpal tunnel c. coracobrachialis d. hook of hate e. mid shaft of the humerus f. supinator canal g. surgical neck of the humerus

The lung apices and cervical pleura extend above the clavicle and first rib through the superior thoracic aperture into the neck. Stab wound would cause tension pneumothorax. -increasing volume of air accumulates within the pleural space, the lungs and mediastinum would deviate contralaterlly. -Signs and symptoms of tension pneumothorax include: -tachycardia, hypotension, tachypnea, hypoxemia, and absence of breath sounds and hyperressonance to percussion of the affected side. Treatment: needle thoracotomy or chest tube a. Spinal Accessory Nerve may be injured during surgrey involving the posterior triangle of the neck ( a region bounded by the SCM, traps and clavicle) b. ansa cerivxclis arises form C1-3 nerve roots and innervates the sternohyoid, sternothyroid and omohyoid muscles of the anterior neck. Penetrating trauma too the neck love the cricoid cartilage can injure this nerve c. carotid body, which contains O2, CO2, and H chemoreceptors, lies at the bifurcation of common carotid artery (just inferior to the hyoid bone). d. the inferior thyroid artery arises from the thyrocervical trunk and courses posterior to the carotid artery and jugular vein to supply the inferior pole of the thyroid gland. Injury to the inferior thyroid artery is commonly associated with coarseness because it runs adjacent to the recurrent laryngeal nerve.

1695 Dude was stabbed above the clavicle, bewttwen the midclavicular and lateral sternal lines. the course of the wound appears to be perpendicular to the skin. Which of the following structures was most likely injured? a. accessory nerve b. ansa cervicalis c. carotid body d. inferior thyroid artery e. lung pleura

Single peak indicates that the population being tested possesses a similar genetic drug metabolizing capacity. A bimodal (discontinuous, polymorphic) curve is from two distinct groups within the study population suggesting a phamacogenetic polymorphism in drug metabolizing capacity. In other words, the two peaks indicate two sets of responder to the drug within the population: -one that rapidly converts the drug into tis metabolite -another that converts the drug slowly, leading to accumulation of the original drug in the plasma Isoniazid is metabolized by acetylation to N-acetyl-isoniazid in the liver by N-acetyl-transferase. First/second peaks correspond to slow/fast acetylators. Slow acetylators of isoniazid also metabolize (acetylate) dapsone, hydrazine, and procainamide slowly causing accumulation of these drugs as well. These patients have increased risk of toxic effects Fast acetylators may require much higher therapeutic doses o achieve a therapeutic effect a. methylation is important drug biotransformation method to consider when prescribing drugs such as azathioprine and 6-mercaptopurine, drugs sued in treatment of some inflammatory disorders of the bowel and skin b. glucuronidation is for conjugating bilirubin. c. hydrolysis occurs with enzymes such as esterase and amides. e. amine oxidation is usually undertaken by monoamine oxidase or by CYP oxidative deamination.

1712 Investigators designed an experiment to assess genetic variability in drug biotransformation. A fixed dose of isoniazid is given to a group of volunteers, and the plasma drug concentration is measured four hours following admin of the drug. Variation (2 curves instead of one bell curve) of which of the following processes provides the explanation for the distribution of plasma drug concentration? a. methylation b. glucuronidation c. hydrolysis d. acetylation e. amine oxidation

Behavioral condition: 1. classical: previous neutral stimulus elicits response after association with a natural, unlearned stimulus. 2. operant: behavior becomes associated with an environmental consequence. a. punishment: making behavior less likely to occur -positive: giving the punishment decreases behavior -negative: removal of reward decreases behavior b. reinforcement: making behavior more likely to occur -positive: giving reward to increase behavior -negative: removal of punishment or consequence increase behavior b. displacement is an immature defense mechanism that involves shifting emotions to a safer alternate object or person that represents the original in some regard. An example is the patient becoming angry with her physician and not expressing her feeling, and then later becoming angry with her roommate instead for no reason e. reaction formation is the transformation of an unacceptable impulse or emotion into its opposite. An example would be this patient acting extremely relaxed around her physician and saying that she finds her appointment to be very soothing to defend against underlying anxiety. f. social learning is a theory of personality development that emphasizes the importance of observing and imitating the behavior, attitudes and emotion reactions of others.

1729 A 20 y.o student has a longstanding fear of physicians. He feels anxious and sweats around his PCP. Her roommate is a nursing student, and the patient has recently begun sweating whenever she sees the roommate's stethoscope lying around their apartment. What is this called? a. classical conditioning b. displacement c. negative reinforcement d. positive reinforcement e reaction formation f. social learning

selective estrogen receptor modulators Drugs: tamoxifen/raloxifene MOA: competitive inhibitor of estrogen binding. Mixed agonist/antagonist action Indications: prevention of breast cancer in high-risk patients. -Tamoxifen: adjacent treatment of breast cancer -Raloxifene: postmenopausal osteoporosis A/Es: hot flashes, venous thromboembolism and endometrial hyeprplaisai and carcinoma (w/ tamoxifen only). Estrogen plays a critical role in bone and reproductive organs, including skeletal homeostasis and proliferation of the endometrium. Because estrogen decreases bone resorption, estrogen def in postmenopausal women increase the risk of osteoporosis. In the uterus, unopposed estrogen exposure (eg, estradiol therapy) canc cause excessive endometrial proliferation resulting in endometrial hyperplasia and cancer. Selective estrogen receptor modulators (eg, raloxifine, tamoxifen) are non steroidal compounds that bind to estrogen receptor and exhibit estrogen antagonist and agonist properties in a tissue specific manner. RALOXIFENE: excellent choice for prevention of breast cancer and osteoporosis because it has estrogen antagonistic activity on breast. Furthermore, raloxifine has estrogen antagonistic activity on uterus and doesn't increase the risk of endometrial cancer. TAMOXIFEN: has a strong estrogen antagonist activity in the brest and is used in the treatment of estrogen receptor positive breast cancer. Although tamoxifen produces estrogen-like effects on bone and can reduce the risk of osteoporosis, it agonist activity on the uterus (eg, increased risk of endometrial yperplasia/cancer) limits its potential in osteoporosis management. a. alendronate is a synthetic bisphosphonate. As a bone resorption inhibitor it prevents and treats osteoporosis in postmenopausal women. Bisphosphonates do not protect against breast cancer. c. leuprolide is a GnRH analog with estrogen agonist properties when administered in pulsatile manner and antagonist properties when administered continuously. Its use as an agonist increases estrogen and protects against osteoporosis. However, it use as an antagonist decreases estrogen, protecting against estrogen receptor psotivie breast cancer but not osteoporosis. d. oral medroxyprogesterone reduces the incidence of endometrial hyperplasia and the risk of endometrial carcinoma in postmenopausal women on estrogen replacement therapy. Prolonged use of intramuscular medroxyprogesterone as a contraceptive is associated with decreased bone mineral density.

1795 A 46 y.o woman has vertebral stress fracture diagnosed via x-ray. She has a history of hypothyroidism for which she takes levothyroxine. The patient's last menstrual period was a year ago Her diet consists mainly of vegetables and fruit, and she takes a daily multivitamin. Her mother died of breast cancer at age 52. The patient is concerned that her family history puts her at risk for breast cancer. X-ray absorptiometry studies demonstrate abnormally decreased bone density in the lumbar vertebrae. Which of the following drugs is the best option for decreasing the risk of both bone fractures and breast cancer in this patient? a. alendronate b. ethanol estradiol c. leuprolide d. medroxyprogesterone e. raloxifene f. tamoxifen

Turner is caused majority by paternal meiotic nondisjunction. The loss of the parental X chromosome in the sperm results in a missing X chromosome in most or all cells. 45,X. The loss of X chromosome results in a missing SHOX gene, which is responsible for long bone growth. Meiotic nondisjunction is also responsible of Klinefelter, trisomies 13, 18 and 21. a. a balanced, reciprocal translocation is clinically silent because there is no excess or shortage of genetic material. b. uniparental disomy occurs when an individual inherits 2 copies of a chromosome form one parent and no copies of the chromosome from the other parent. -Prader-Willi: 2 maternal copies of chromosome 15 d/t deleted or unexpressed paternal chromosome. -Angelman syndrome: 2 paternal copies of chromosome 15 d/t deleted or unexpressed maternal chromosome.

1810 A girl has turner syndrome. What is the underlying mechanism? a. balanced translocation b. frameshift mutation c. meiotic nondisjunction d. trinucleotide repeat expansion e. uniparental disomy

Antiemetic drugs clinical uses: 1. motion sickness/hyperemesis gravid arum (promethazine) a. antimuscarinics - scopolamine b. antihistamines - diphenhydramine, meclizine, promethazine 2. Chemotherapy induced emesis a. dopamine receptor antagonist - prochlorperazine, metoclopramide b. 5-HT3 receptor antagonists - ondensetron and granisetron c. neurokinin 1 (NK1) receptor antagonist: aprepitant, fosaprepitant Vomiting reflex: 1. area postrema (4th ventricle) has chemoreceptor trigger zone that can respond to many neurotransmitters, drugs or toxins. 2. nucleus tracts solitaires (NTS) in medulla. Receives information from the area postrema, GI via the vagus nerve, vestibular system and CNS (eg, meninges, hypothalamus). Neurons from the NTS project to other medullary nuclei and coordinate the vomitting process. The 5 major receptors involved in stimulating the vomiting reflex in the area postrema and adjacent vomiting center nuclei are M1, D2, H1, t-HT3, serotonergic and NK1. 5-HT3 receptor antagonists (eg, ondansetron, granisetron, dolasetron) are highly effective in preventing chemotherapy-inducedvomiting. 2 primary MOAs: 1. blocking vagus-mediated nausea and vomiting 2. blocking serotonin receptors in the area postrema and NTS Other shit you can use is: -NK1 receptor antagonists (eg, aprepitant), which inhibit substance P and help prevent both acute vomitinga nd delayed emesis -Dopamine receptor antagonists (eg, metoclopramide), associated with drowsiness and dystonic reactions a/d: histamine blockers and antimuscarinic/anticholinergic agents are most helpful for vestibular nausea and vomiting. b. motion regulates inter digestive migrating contractions. Erythromycin is a motion receptor agonist used for gastroparesis c. Mu opioid receptor agonists (eg, morphine) are useful for cancer related pain control but often have side effects such as nausea and vomiting.

1817 A 64 y.o woman has intractable nausea and vomiting. She started chemotherapy 1 week ago. Which of the following agents would be dost helpful in treating this patient's symptoms? a. H1 blocker b. motilin receptor agonist c. Mu opioid receptor agonist d. M1 receptor antagonist e. serotonin 5-HT3 receptor antagonist

This kid has HUS Etiology: E. coli O157:H57 or Shigella dysenteriae. Path: Verotoxin induces endothelial damage = platelet activation = formation of micro thrombi = platelet consumption = thrombocytopenia. Micro thrombi also damage RBCs = hemolytic anemia Presentation: -Anemia (eg, pallor, weakness, tachycardia) -Acute kidney injury (eg, oliguria/anuria, increased creatinine) Labs -Decreased hemoglobin and haptoglobin -Increased lactate dehydrogenase and unconjugated bilirubin -Bleeding time elevated (however PT and PTT not affected because there are clotting factor def or DIC in HUS) e. Coombs test, or direct anti globulin test, detects autoantibodies against RBCs if they are present in the serum. Hemolytic anemia in HUS is a mechanical problem in stead of an autoimmune one f. streptozyme detects antibodies against Progenies and can be used to retrospectively diagnose strep infections (eg, when evaluating PSGN). However, PSGN will have edema and HTN, but no hemolytic anemia.

1851 A 5 y.o boy has 2 day history of dark, low-volume urine and decreased energy. Four days before these symptoms started he had pain, fever and bloody diarrhea, which he most likely had from a hamburger he ate at a family picnic. Vital signs are within normal limits. PE shows pallor. Labs shows anemia, thrombocytopenia and elevated BUN and creatinine. Which of the following findings is most likely to be seen in this patient? a. elevated haptoglobin level b. elevated serum indirect bilirubin c. elevated PT and PTT d. low fibrinogen and elevated D-dimer level e. positive Coombs test f. positive streptozyme test

Kid has complete AV canal defect, most common type of cardiac defect seen in Down syndrome. Failure of endocardial cushion results in osmium primum atrial defect: a ventricular septal defect; and a single AV valve. Significant left-to-right shunting an dAV valve reguregurgitation lead to excessive pulmonary blood flow and symptoms of heart failure (eg, tachypnea, poor feeding) Auscultatory findings of AV valve regurgitation (holsystolic, best heard at the apex) and increased pulmonary venous return (mid diastolic rumba) are characteristic. a. DiGorge is characterized by thyme aplasia (T cell def) and hypoparathyroidism (hypocalcemia). it is associated with tetralogy of Fallot, trunks arterioles and transposition of the great arteries. c. Marfan syndrome (fibrillin-1 mutation) is associated with cystic medial necrosis of the aorta, which may result in dissecting aortic aneurysms and aortic valve regurgitation. Mitral valve prolapse is also common, but septal defects are not d. mutations of frataxin, a mitochondrial protein important in iron homeostasis and respiratory function, cause Freidrecih ataxia. It is characterized by spinocerebellar degernation and is associated with hypertrophic cardiomyopathy, but not septal defects. e. mutations in tubers and harmartin are seen in tuberous sclerosis. These patients may develop cardiac rhabdomyomas in ventricular walls and AV valves, cutaneous angiofibromas (adenoma sebaceous), CNS hamartomas and renal cysts. f. turner is associated with bicuspid aortic valve (most common cardiac lesion) and coarctation of the aorta.

188 A 5 w.o boy has history of rapid breathing and tiring with feeds. CS exam shows hyper dynamic precordium, a mid diastolic rumba at the left sternal border, and a 3/6 holosystolic murmur in the apex that radiates to the left axilla. ECG shows sects in the lower part of the interatrial septum and the inter ventricular septum. What condition does he dos likely have? a. 22q11.2 deletion b. autosomal trisomy c. fibrillar mutation d. frataxin mutation e. hamartin gene mutation f. sex chromosome monosomy

This medication inhibits neprilysin, a metalloprotease that inactivates several peptide hormones, including bradykinin, glucagon, enkephalins and natriuretic peptides. ANP is made by atrial cardiomyocytes in response to atrial stretch, which is indicative of systemic volume expansion. -Lower blood pressure through peripheral vasodilation, natriuresis and diuresis. -uses cGMP Organs affected are: 1. kidney: dilates afferent arterioles = increase GFR and urinary excretion of Na and H20. -it also limits Na reabsorption (in PCT and inner medullary of th recollecting duct) -inhibits renin secretion 2. Adrenal gland: restrict aldosterone secretion, leading to an increase in Na and H2O excretion by kidneys 3. blood vessels: ANP relaxes vascular smooth muscle in arterioles and venues, producing vasodilation. It also increases capillary permeability, leading to fluid extravasation into the interstitium and a decrease in circulating blood volume. b. duodenal mucosal cells produce many GI hormones such as gastrin, secretin and CCK. But none of them increases urinary output or decreases TPR c. glomerular zone of the adrenal gland secretes aldosterone. Which would increase H2O and Na reabsorption causing the extracellular fluid to expand and BP to increase d. oxytocin and ADH are 2 hormones produced by the posterior pituitary gland. Vasopressin increases water permeability of the renal collecting ducts, thereby increasing water reabsorption. Oxytocin stimulates uterine contractions in late pregnancy and facilitates breast milk ejection postpartum e. JG cells secrete renin, which catalyzes the formation of angiotensin I from angiotensinogen. Angiotensin I is converted in the lungs into angiotensin II , which causes vastconstriction and aldosterone please.

1978 A 44 y.o man has nonischemic cardiomyopathy and chronic systolic heart failure. The patient is now having worsening of dyspnea on exertion, and now has to stop halfway to catch his breath while climbing the stairs. He was given a neutral endopeptidase inhibitor. The med prolongs the action of an endogenous polypeptide hormone that increases urinary output and decreases total peripheral vascular resistance. This polypeptide hormone is most likely produced by which of the following tissues? a. atrial cardiomyocytes b. duodenal epithelium c. glomerular zone of the adrenal gland d. posterior pituitary gland e. renal jg cells

DKA causes anion gap metabolic acidosis. -Patients with metabolic acidosis usually develop a compensatory respiratory alkalosis by hyperventilating (eg, Kussmaul respirations) causing his PaCO2 to drop. -Severe DKA can cause pulmonary edema or significantly decreased mental status, which may lead to subsequent RESPIRATORY FAILURE. If PaCO2 persists above the expected compensatory range (PaCO2 = (1.5 x HCO3) + 8 +/-2), it indicates a superimposed respiratory acidosis interfering with the normal compensatory response. This patient's HCO 3 is 12, his PaCO2 should be in the rang elf 24-28 mm Hg. The fact that his PaCO2 is much higher than expected indicates that he has a mixed acid-base disturbance consisting of metabolic and respiratory acidoses. b. dude has DKA. His tubules should be secreting more H and increase reabsorption of HCO3 d. renal tubular necrosis is characterized by progressively increasing serum creatinine with epithelial cell cars and muddy brown granular casts seen on urinalysis. Acute tubular necrosis is unlikely in this patient with normal serum creatinine.

1979 Kid has DKA. pH = 7.27 paCO2 = 40 mm Hg HCO3 = 12 Glucose = 498 Normal creatinine Which of the following is most likely occurring in this patient? a. adrenal insufficiency b. increased urinary bicarb loss c. low plasma renin activity d. renal tubular necrosis e. respiratory failure

Most common intra-abdominal organ injured during blunt trauma is the spleen. It is an intraperitoneal organ derived from the mesoderm of the dorsal mesentery. -It is unique in that its blood supply derives from a forgot derivative (splenic arty comes off of the celiac trunk). -Venous return courses through the splenic vein to return to the portal circulation rather than the systemic circulation a. liver is an endoderm-derived structure of the forgot and receives its arterial blood supply from the proper hepatic artery (via the celiac trunk). b. kidneys are derived from the mesoderm. However the kidneys are retroperitoneal and supplied by the renal artery (from the abdominal aorta) and rated by the renal veins (to the inferior vena cava) c. First 2/3 of the transverse colon is derived from the midgut and supplied by the superior mesenteric artery. the last 1/3 of the transverse colon arises from the handgun and is supplied by branches of the IMA. These 2 vessels anastomose via the marginal artery d. the pancreas is an endoderm-derived structure of the forgot that receives blood primarily from the superior and inferior pancreaticoduodenal arteries.

2022 A 35 y.o man has persistent abdominal pain. Yesterday he suffered a full speed collision with another player during a football game. It is determined that his injury involves an organ that is supplied mainly by an artery of the forgot even though the organ itself is not a forgot derivative. Which of the following organs is most likely injured? a. liver b. kidney c. colon d. pancreas e. spleen

Genetic information flows from DNA to RNA to proteins. Post uekaryotic DNA sequences consist of coding exon, noncoding introns, and 2 promoter regions ( the CAAT box and the TATA box). The CAAt box is located 70-80 bases upstream of the beginning (5' end) of the coding region, and the TATA box is located 25 bases upstream from the beginning of the coding region. Gene transcription begins when RNA polymerase II attaches to one of the promoter regions in a process that requires general transcription factors. A DNA enhancer region then binds activator proteins that associate with transcription factors and RNA polymerase II at the promoter, thereby increasing gene expression. Although promoters are not directly translated into protein, promoter mutations can cause abnormal gene expression by altering ability of RNA polymerase II and transcription factors to bind. a. DNa methylation is part of the epigenetic code. This process sis carried out by DNA methyltransferases and serves to silence the genes it affects b. folding of a formed polypeptide into its secondary adntertiary structures is entirely spontaneous and is determined by the amino acid sequence in the protein's primary structure. heat shock proteins assist in the spontaneous refolding of proteins. c. Posttranscriptional RNA splicing is facilitated by snRNPs that remove introns from heterogeneous nuclear RNA (hnRNA) containing GU at the 5' splice site and AG at the 3' splice site d. the TATA box only participates in the initiation of transcription. The addition of nucleotides to th forming RNA molecule (RNA elongation) continues until RNA polymerase II encounters a termination signal. f. In eukaryotes, translation initiation requires both ribosomal subunits (60S and 40S) with their associated rRNA, mRNA, initiation factors, initiator tRNA charged with methionine, and GTP. The assembled ribosome then recognizes the AUG start codon on mRNA to being the process

2030 A genetic study performed on a 10 y.o boy shows a single base substitution mutation from TATAAA into TAGAAA (-25 bp upstream of the transcription start start site). This mutation is most likely to affect which of the following processes? a. DNA methylation b. polypeptide folding following translation c. post transcriptional RNA splicing d. RNA elongation e. transcription initiation f. translation initiation

A-T G-C In this experiment, the repeated deoxythymidine on the latex beads most likely bind to the poly A tail of a mature mRNA. Post-strancription modification of mRNA is as follows: 1. 5' capping: a 7-methyl-guanosine cap is added to the 5' end of the mRNA 2. Polyadenylation: poly-A tail is added to most eukaryotic mRNA, which are not transcribed from the DNA template, but instead, has a consensus sequence (AAUAAA) located near the 3' end of the RNA molecule directs the addition of poly-A tail. This trial protects the mRNA from degradation within the cytoplasm after it exits the nucleus. 3. Splicing. The initial pre-MRNA contains sequence from coding and noncoding regions of DNA, aka exons and introns, respectively. SPliceosomes (complex of snRNPs and other shits) remove introns containing GU at the 5' splice site and AG at the 3' splice site. a. aminoacyl-tRNA charged with its amino acid. the clover leaf structure of tRNA consist of a 3'CCA tail (amino acid binding site); a T loop abundant in ribothymidine, pseudouridine and cytidine residues; a D loop rich in dihydrouride residues and an anticodon loop. c. DNA promoter regions help initiate transcription by binding transcription factors and RNA pol II. Promoter regions contain consensus sequences that are typically AT-rich (eg, TATA and CAAT boxes) or GC-rich (eg, GC box) d. rRNA is a component of the ribosome that catalyzes peptide bond formation during translation f. telomeres are located at the ends of chromosomes and contain TTAGGG repeats, which are added by the enzyme telomerase (RNA-dependent DNA polymerase). Craig chsortening in tolemere length is though to signal programmed cell death.

2033 In an experiment, cluttered fibroblast are mechanically lysed, and the membrane lipids and cellular proteins are chemically removed to isolate nucleic acids. The cellular extract containing the purified nucleic acids is incubated along with short sequences of repeated deoxythymidine residues fixed to latex beads. The solution is washed several times to remove unbound molecules. Which of the following types of nucleic acid is most likely bind the strongest to the latex beads in this experiment? a. Aminoacyl-tRNA b. mature mRNA c. promoter regions of DNA d. ribosomal RNA e. splice sites of pre-mRNA f. telomere regions of chromosomes

After transcription, the preliminary, unprocessed mRNA is known as precursor mRNA, or heterogenous nuclear RNA (hnRNA). Eukaryotic pre-mRNA undergoes significant post transcriptional processing before leaving the nucleus, including 5' capping, poly A tail addition, and intron splicing. Once mRNA is finalized, it leaves the nucleus bound to specific packaging proteins. Upon entering the cytoplasm, these mRNA complexes often associated with ribosome to undergo translation. However, certain mRNA sequences instead associate with proteins that are found in P bodies. P bodies are distinct foci within eukaryotic cells that are involved in MRNA regulation and turnover. They play a fundamental role in translation repression and mRNA decay, and captain numerous proteins including gRNA exonuycleases, mRNA recapping enzymes, and constituents involved in mRNA quality control and microRNA-induced mRNA silencing. P bodies also seem to function as a form of mRNA storage, as certain mRNAs are incorporated into P bodies only to be alter related and utilized for protein translation. a/b: The 4' end of all mRNA is capped with a 7-methylguanosine residue by a unique 5' to 5' linkage, which occurs in two stages. The first step is addition of guanine triphosphate to the 5' end of mRNA in reaction catalyzed by guanylytransferase. Methylation of the guanosine cap is then catalyzed by guanine-7-methyltransferase. Capping of the precursor RNA occurs in the nucleus as the ran is being transcribed. This methylated cap protect mRNA from degradation by cellular exonuclease, and allows it to exit the nucleus. c. mRNA is polyadenylated at the 3' end by the polyadenylate polymerase complex, which recognizes a specific sequence (AAUAAA), cleaves the pre-mRNA molecule a few residues downstream from the sequence, and then adds a stretch of 20-250 adenine residue called the poly A tail. The addition of the poly A tail occurs before th emRNA exits the nucleus. IN the cytosol, th poly A tail is gradually shortened, eventually leading to mRNA degradation. d/e: since pre-mRNA contains both introns and exon, and only exon code for proteins, introns must be excised before translation through process known as splicing. Splicing of pre-mRNA occurs within the nucleus and is facilitated by interaction of pre-mRNA with small ribonucleoprotein particles called snRNPs (or "slurps" for short).

2035 A 32 y.o man is recovering from extensive burns. Fibroblast near the site of junury actively synthesize precursor mRNA to be used as templates for protein synthesis. After transcription, extensive processing of the precursor RNA occurs to form the finalized mRNA sequence. The finalized mRNA then exits the nucleus and undergoes translation by ribosome complexes before being degraded. Which of the following steps involving the processing and handling of mRNA occurs only within the cytoplasms f cells? a. 5'-terminal guanosine triphosphate addition b. methylation of the 5'-temrinal guanine c. multiple adenine nucleotide attachment to the 3'-end d. interaction with snRNP e. removal of intervening sequences f. interaction with P bodies

Following transcription, pre-mRNA is the initial transcript that contains both intron and exon sequences. Before leaving the nucleus, pre-mRNA must be processed to mature mRNA by: 1. 5' methylguanosinn capping 2. 3' polyadenylation 3. splicing Splicing is performed by spliceososomes, which are complexes of snRNPs. Spliceosomes remove introns containing GU at the 5' splice site and AG at the 3' splice site. The freed 3'OH of exon 1 then forms a phophodiester bond with the 5'-phosphate at the splice acceptor site, joining exon 1 and 2. Mutation at splice sites may result in inappropriate removal of eons and retention of introns. This often leads to the formation of proteins with impaired structure and function. a. Polyadenylation of the 3' end of mRNA is performed by the enzyme polyadenylate polymerase. This process stabilizes the mRNA and helps it exit the nucleus. b/d. In eukaryotes, translation is initiated when the small ribosomal subunit attaches to the 5' cap of mRNA and then scans for the AUG start codon within the Kozak consensus sequence. The 5' cap also protect against exonuclease and helps stabilize mRNA in the cytosol e. Termination of polypeptide synthesis occurs at the 3stop codons (UAA, UAG, UGA) in mRNA. Mutations in stop codons (nonstop mutations) can result in continued and inappropriate translation of mRNA into the 3'-untranslated region, producing an extremely long, nonfunctional polypeptide.

2038 An 8 y.o boy of Ashkenazi Jewish ancestry is brought to the office after developing reduced sensitivity to pain, impaired tear formation, and orthostatic hypotension. Familial dysautonomia is suspected d/t the patient's symptoms and heritage. This disorder is caused by loss of function of the IKAP protein, which is essential for development and survival of sensory and autonomic neurons. IKAP gene sequencing reveals a single nucleotide substitution that causes a guanine residue to be replaced by adenine at the highlighted position in the normal gene sequence shown where there is a mutation to the guanine amino acid of an intron. Which of the following is the most likely effect of this mutation? a. decreased mRNA export to the cytosol b. impaired ribosomal attachment to mRNA c. incorrect splicing of pre-mRNA d. increased degradation of mRNA by 5' exonuclease e. translation of the 3' untranslated region of mRNA

Patient has chronic severe aortic regurgitation. Failure of the aortic valve leaflets to remain closed during diastole results in a portion of left ventricular stroke volume to lead from the aorta back into the left ventricle. The regurgitant blood flow increases left ventricular end diastolic volume (preload) and wall stress, with resultant eccentric hypertrophy. The gradual increase in LV chamber size d/t eccentric hypertrophy increases stroke volume and maintains CO. In contrast, patients with acute AR have a small LV cavity and cannot significantly increase forward stroke volume to accommodate for the regurgitant blood flow. This leads to a decline in cardiac output, along with a rapid increase in left ventricular end diastolic pressure, leading to acute pulmonary edema. b. a decrease in aortic elasticity (i.e., increased aortic stiffening) increases LV after load and leads to reduced CO d. rapid regurgitation of blood back into LV leads to a sudden fall in aortic pressure during diastole, resulting in a wide pule pressure in patents with chronic severe AR. This leads to an overall decrease in LV after load. f. CO in patients with chronic AR is maintained primarily by increase in LV stroke volume. However, in acute aortic regurgitation, eccentric hypertrophy doesn't have time to develop and the potential increase in LV stroke volume is limited. The resulting decrease in cardiac output triggers a reflex increase in HR.

237 A 44 y.o IV drug user is having S. aureus infective endocarditis. He was successfully treated with long course of antibiotics. 3 months later, his ECG shows severe aortic regurgitation as a sequela to the prior infection. Which of the following changes is most likely responsible for imitating cardiac output in the setting of this valvular abnormality? a. concentric left ventricular hypertrophy b. decrease in aortic electricity c. decrease in left ventricular preload d increase LV after load e. increase LV stroke volume f. sustained increase HR

Patient has aortic regurgitation: repetitive, widened pulse pressure together with unusually large LV stroke volumes and heart murmur. Pathophysiology: 1) nocturnal palpitations: forceful ventricular contractions ejecting large SV. 2) Head pounding: unusually high amplitude pulsations of the intracranial arteries with each heartbeat. 3) involuntary head bobbing: sign of widened pulse pressure (pulse pressure = peak systolic arterial pressure - end diastolic arterial pressure). a. restricted LV filling would reduced LV EDV aka reduced preload causing reduce stroke volume, which would lower pulse pressure b. Impaired LV contractility cause a reduction in stroke volume for a given reload, resulting in lower pulse pressures. c. LV outflow tract obstruction, as can result form aortic stenosis or hypertrophic cardiomyopathy, could cause a murmur but would tend to reduce SV and thus PP d. combined systolic and diastolic HTN are not necessarily accompanied by an abnormally large pulse pressure or murmur

238 A 34 y.o South Asia immigrant has heart palpitations that are particularly prominent at night. He also notes that with moderate exertion, he experiences head "pounding" accompanied by involuntary head bobbing. He remembers being diagnosed with a heart murmur years before, but he cannot recall the type and has never received any treatment. Based on this patient's history you suspect? a. restricted LV filling b. impaired LV contractility c. LV outflow obstruction d. systolic diastolic HTN e. widening of the pulse pressure

Lamotoigine is an anticonvulsant MOA: blocks voltage-gated Na channels. Indicated to treat partial and generalized seizures and is also effective with bipolar disorder. Side effects: -10% has Stevens-Johnson syndrome or toxic epidermal necrolysis. These two are at different endpoints along the same disease continuum: SJS <10% of the body surface and TEN >30%. Symptoms: 1. flu-like symptoms (eg, fever, malaise, myalgia) followed by cutaneous and mucosal lesion. 2. Histo: epidermal necrosis and suepidermal bullae. Other anticonvulsants associated with SJS?TEN include carbamazepine, phenobarbital and phenytoin.

356 A 14 y.o boy has seizures characterized as blank stare and lip smacking, followed by generalized shaking. He bites his tongue during the episodes and feels confused afterwards. His seizures have been refractory to various anti epileptic medications and he is going to be started on lamotrigine. What is the most serious side effect? a. abdominal pain b. cough and fever c. diarrhea d. heartburn e. skin rash f. urine discoloration

By the terminal bronchioles, the airway epithelium has changed from pseudo stratified ciliated columnar to dilated simple cuboidal. Cilia are present through the respiratory bronchioles, but are not present in the alveolar ducts or in the alveoli themselves. This arrangement is important because cilia function to propel mucus toward the pharynx. The rpecense of cilia beyond the most distal mucus-producing cells (goblet cells and in the smallest bronchi, just before the bronchioles begin), prevents bronchiolar mucus accumulation and airflow obstruction. c/d/e: the submucosal mucous and serous glands travel with the cartilaginous plates in the tracheobronchial tree. These elements end at the termination of the smallest bronchi. Bronchioles generally lawck goblet cells, glands and cartilage

480 Which of the following features is last to disappear as the epitheliumchaiges long the respiratory tube? a. cilia b. goblet cells c. mucous glands d. serous glands e. cartilage

this patient's clinical findings are highly suggestive of Klinefelter syndrome, the most common cause of male hypogonadism. In klinefelter syndrome, progressive destruction and hyalinization of the seminiferous tubules cause the testes to be small and firm. -Sertoli cells are damaged causing inhibin levels to be low -Leydig cells are usually dysfunctional as well thus patient will have low level of testosterone -The loss of feedback leads to elevated FSH and LH. Patients can also develop high estrogen levels and gynecomastia d/t increased aromatase activity (stipulated by gonadotropin excess). Most patients with classic (47, XXY) Liefelter syndrome have azoospermia and are infertile, but those with mosaic variants may have variable degrees of spermatogenesis. a. Low LH and FSH, along with low testosterone, are consistent with hypogonadotropic (central) hypogonadism, which can result form damage to the hypothalamus or pituitary gland The congenital def of GnRH, called Kallmann syndrome, is generally associated with anosmia. In addition, hypogonadotropic hypogonadism is less likely to cause gynecomastia as the decrease in gonadotropins reduces aromatase activity. b. patient with normal hormonal profile but lacking sperm may have an obstruction somewhere along the path from the testes to the seminal fluid. For example, this occurs with congenital absence of the vas deferent in cystic fibrosis. c. decreases LH , normal FSH and elevated testosterone in setting of a low sperm count is suggestive of exogenous testosterone use. High androgen levels suppress LH secretion,d decreasing endogenous testosterone production. Despite the high circulating levels of exogenous androgens, these patients have a low sperm count as local androgen contractions in the seminiferous tubules are suboptimal for spermatogenesis. Over time, this leads to atrophy of the seminiferous tubules and a decrease in testicular size. e. normal LH and testosterone, elevated FSH and a low perm count can be seen in patients with cryptorchidism. In this condition, the seminiferous tubules are damaged (resulting in elevated FSH levels) and the interstitial Leydig cells are preserved (maintaining normal LH and testosterone levels).

582 A 27 y.o man is being evaluated for infertility. On PE, he has bilateral gynecomastia and small, from testes. His lower extremities are abnormally long. Further evaluation will most likely show which of the hormonal patterns of LH, FSH, testosterone and sperm count?

Symptoms: -anxiety, insomnia -palpitations -heat intolerance -increased perspiration -weight loss without decreased appetite -goiter PE -HTN -Tremors involving fingers/hands -hyperreflexia -proximal muscle weakness -lid lag -afib There are many causes, however Graves disease has very specific symptoms that are not commonly seen in other types of hyperthyroidism, which are: 1. infiltrative dermopathy (pretrial myxedema) 2. exophthalmos 3. periorbital edema 4. eye movement limitations -These are specific because hyperthyroidism is d/t an autoimmune response directed against TSH receptor. And other similar receptors that is widely distributed throughout the body, particularly on adipocytes and fibroblasts. -These patients develop lymphocytic infiltration of the orbital and pretrial connective tissue d/t increased TSH receptor expression in these regions. Cytokines released by activated T cells increase fibroblast proliferation and secretion of glycosaminoglycans, resulting in mutinous edema and tissue expansion. All other choices are not symptoms specific to any particular cause of hyperthyroidism. -afib is a complication of hyperthyroidism -you would have heat intolerance secondary to increased metabolic rate -thyrotoxicosis can cause increase ALP d/t increase bone turn over -hand tremors are d/t sympathetic overactivity of hyperthyroidism

623 Which of the following findings would be most suggestive of a specific underlying cause of a patient hyperthyroidism? a. afib b. heat intolerance c. increased serum ALP d. lower leg skin thickening and induration e. tremor of the outstretched hands

Primary hyperPTH classic symptoms: -bone pain -renal stones -GI disturbances (eg, peptic ulcer disease) -psychiatric disorders "bones, stones, abdominal groans and psychic moans." asymptomatic hypercalcemia is the most common presentation nowadays d/t routine measurement of calcium in the chemistry profile. Pathology: 1. PTH increases renal tubular Calcium reabsorption (although most patients have hypercalciuria). 2. increased production of calcitriol (1,25 dihydroxyvitamin D) in the kidneys causing increased GI calcium absorption. 3. PTH mediated increase in bone resorption (via osteoclast activation) Labs -Hypophosphatemia as PHP decreases phosphate resorption in the proximal renal tubules. Morphology, skeletal finding -involves cortical (compact) bone in the appendicular skeleton ( the pectoral girdle, pelvic girdle and limbs). -subperiosteal erosions in phalanges of the hand, a granular "salt and pepper" skull, and osteolytic cystic the long bones a disorganized lamellar bone structure in a mosaic pattern = Paget disease. Serum calcium and phosphorous are normal. b. osteoid matrix accumulation around traveculae is seen in osteomalacia. Hits shows excessive unmineralized osteoid with widened osteoid seams. patients will typically have low urinary calcium. c. persistence of the primary spongiosa in the medullary cavity with no mature trabeculae is a classic fining of osteoporosis (i.e., "marble bone disease"). Osteoporosis is caused by decreased osteoclastic bone resorption, which results in accumulation of woven bone and diffuse skeletal thickening e. trabecular thinning with fear interconnections is characteristics of osteoporosis. Although longstanding PHP causes thinning of cortical bone, the trabecular architecture remains relatively preserved.

631 A 46 y.o woman has recurrent renal colic. PMH is peptic ulcer in which she takes famotidine daily. Labs shows high urinary calcium and low phosphorus. Which of the following changes in bone structure is most likely associated in this patient's condition? a. lamellar bone structure resembling a mosaic pattern b. osteoid matrix accumulation around trabecular c. osteoid matrix accumulation around trabecular c. spongiosa filling medullary canals with no mature trabecular d. subperiosteal resorption with cystic degeneration e. trabecular thinning with fewer interconnections.

Causes of ostemelitis: Childhood age: hematogenous seeing during an episode of bacteremia. Most frequent pathogen is S. aureus. Typical location is long bones. Sickle cell disease: Hematogenous seeding to infarcted bone. Most frequent pathogens are Salmonella and S. aureus. Typical location is long bones. Post disease: hematogenous seeding from lungs. Most frequent pathogen is mycobacterium tuberculosis. Typical location vertebrae. DM: contiguous spread from infected foot ulcer. Most frequent path is poly micro affecting the bones of the feet Recumbent patients with impaired mobility. Contiguous spread from pressure sores. Most frequent pathogen polymicrobial affecting the sacrum and heels Recent trauma or orthopedic surgery. Direct inoculation. Polymicrobial and location is variable. Osteomyelitis is an infection of the bone and bone marrow that occurs by 1 of 3 mechanisms: 1. hematogenous seeding d/t an episode of bacteremia 2. spread from a contiguous focus infection, as occurs in an infected diabetic foot wound 3. direct inoculation of bone, such as with a compound fracture Hematogenous osteomyelitis occurs predominantly in children and most frequently affects the long bones the tibia, fibula, and femur are most often involved. Adults who develop the condone are more likely to have vertebral involvement and frequently have a predisposition to bacteremia d/t risk factors such as IV drug abuse or indwelling vascular catheters. S. aureus is implicated in most cases of acute hematogenous osteomyelitis in otherwise healthy children a. enterococcus faecalis causes a variety of infections, including endocarditis, meningitis, and urinary tract infections. Enterococcus can cause vertebral osteomyelitis after a recent UTI via bacteremic spread. b. moraxella catarrhalis is part of the normal flora of the URT. It causes otitis media and sinusitis in healthy individuals and is frequently responsible for causing exacerbation of COPD. d. S. epidermis is ubiquitous in nature and is commonly isolated in cultures as contaminant. However,r S. epidermis can also be pathogenic, colonizing IV catheters and the foreign bodies such as prosthetic heart valves and orthopedic hardware, leading to bacteremia and sepsis. e. S. agalactiae (group B strep) frequently colonizes the GI and urogenital tracts. Infants porin vaginally to colonized mothers can develop serious neonatal infections, including sepsis, pneumonia, and meningitis. For this reason, pregnant women testing positive for group B strep are treated with antibiotic prophylaxis during labor and delivery. f. s. pneumoniae is the most common etiologic agent of community acquired pneumonia. It also causes otitis media in children, sinusitis, meningitis and sepsis g. S. aureus, S. pyogenes (group A strep) is the second most common cause of hematogenous osteomyelitis in children. Group A strep are also responsible for strep pharyngitis and skin infections such as impetigo and necrotizing fasciitis.

646 A 10 y.o boys has high fevers and chills for the last few days. He also complains of dull pain just above the left knee. PE shows point tenderness 3 cm above the kneecap. There is no joint effusion. Radiographs show soft tissue swelling and periosteal reaction over the lower end of the femur. Which of the following organisms is most likely responsible for this patient's symptoms? a. enterococcus faecalis b. moraxella catarrhalis c. s. aureus d. s. epidermidids e. s. agalactiae f. s. pneumoniae g. s. pyogenes

Patient has acute angle closure glaucoma and has been treated with acetazolamide, a diuretic that works by inhibiting carbonic anhydrase, which is found in high concentrations in the proximal tubule and is responsible for catalyzing reactions necessary for NaHCO3 reabsorption. By inhibiting carbonic anhydrase, acetazolamide and other carbonic anhydrase inhibitor effectively block HCO3 reabsorption in the proximal tubules resulting in the enhancement HCO3 and water excretion as well as increased urinary pH and potential metabolic acidosis Carbonic anhydrase is also present in the eyes, pancreas, GIT, CNS and RBCs. In the eye, carbonic anhydrase modulates HCO3 formation in the aqueous humor. Inhibition of carbonic anhydrase will decrease HCO3 and aqueous humor formation; thus, a number of carbonic anhydrase inhibitors are used to relieve intraocular pressures in open angle and angle closure glaucoma. Common side effects of carbonic anhydrase inhibitors include somnolence, paresthesias and urine alkalization. Rare side effects include metabolic acidosis, dehydration, hypokalemia and hyponatremia. Loop diuretic work in the thick ascending limb and is the most potent diuretic DCT actively transport Na and Cl and is impermeable to water. Thiazide works here. Collecting duct system includes the collecting tubules and ducts. Here, aldosterone and ADH make final adjustment to eye trolleys and water content. Potassium-sparing diuretics and aldosterone antagonists work in the collecting duct

682 A 5.7 y.o man has severe right sided see pain and ipsilateral headache. Furthermore, the patient reports severe nausea and describes seeing "halos" around objects. After initial treatment with the appropriate medication, the severity of his pain decreases. he also experiences increased diuresis with highly alkaline urine. the drug used to treat this patient's eye condition predominantly acts on what nephron segments?

Acute hemolytic transfusion reaction -fever/chills -dyspnea -hypotension -chest and/or back pain -hemoglobinuria Complications: DIC and renal failure Acute onset (minutes to hours) d/t ABO incompatibility. Immediate cessation of transfusion. Type II hypersensitivity reaction: -anti-ABO antibodies (mainly IgM) binds to antigens on transfused blood = complement activation -anaphylatoxins (C3a and C5a) cause vasodilatation and symptoms of shock. 0membrane attack complex (C5b-C9) leads to complement-mediated cell lysis. a. cytotoxic CD8 T-lymph destroyed intracellular pathogens. Activates by binding to antigen-MHC class I complex. Causes apoptosis of infected cell c. TNF-alpha is produced by macrophages in response to bacterial endotoxin leading to septic shock (eg, fever, hypotension and tachycardia) when released in large amounts. However, this doesn't cause hemoglobinuria. d. IgE = type I hypersensitivity. Causes allergic transfusion reaction. Can be mild (urticaria, itching) or result in severe anaphylaxis (IgA deficient patients with anti-IgA antibodies) e. Type III is vascular deposition of immune complexes. This occurs in SLE and RA, where antigen-antibody complexes causes tissue inflammation and destruction

747 Dude was in a car accident. He is bleeding from several sites and is hypotensive. He was given blood transfusion. After a few minutes, he started having difficulty breathing, chills and pain in the chest and back. He urine bag is now brown. Which of the following is most likely cause of this patient's new findings? a. CD8 lymphocyte mediated cytotoxicity b. complement mediated cell lysis c. endotoxin induced TNF-alpha surge d. IgE mediated reaction to serum proteins e. vascular deposition of immune complexes

Dude has acne, but he is past the pubertal androgen surge. However, he is an aspiring professional athlete. He might be taking anabolic steroid (eg, methyl testosterone). Other drugs that can cause acne are epidermal growth factor inhibitor and lithium Four major elements underlying formation of acne: 1. follicular epidermal hyper proliferation 2. Excessive sebum production 3. inflammation 4. P. acnes After conversion of dihydrotestosterone, androgen promote follicular epidermal hyper proliferation and excessive sebum production. a. ephedrine is a sympathomimetic drug similar to amphetamine. Use to treat bronchospasm and as a stimulant to suppress appetite and a decongestant. d. UV exposure predisposes to skin cancer and exacerbates some dermatoses (eg, lupus, dermatomyositis, pellagra, polymorphous light eruption), but doesn't cause acne. e. excessive sweating can exacerbates acne but doesn't cause it

7585 A 26 y.o male has persistent skin rash. He is a minor league baseball player and is outside all the time. He chews tobacco drinks alcohol on weekends. PE reveals a comedonal (small pimples) and inflammatory nodular eruption on the patient's face, chest and back. Which of the following the most likely cause of this patient's skin findings? a. ephedrine b. methyl testosterone c. chewing tobacco d. UV e. excessive perspiration

During the first encounter with an antigen (allergen), antigen-presenting cells such as macrophages, B lymph and dendritic cells take up and process the antigen and display it in associated with MHC II molecules. T-cells become activated when they detect the MHC II-associated antigen, and differentiate into Th1 or Th2 subsets. Th2 lymph promote the humoral immune response by secreting IL-4 and IL-5: Some activated B-cels transform into plasma cells that produce antigen-specific IgM (primary response). The majority of activated B-cells proliferate in the germinal centers of lymph nodes an transform into memory cells. Antibody switching then allows B-cells to secrete other amor subtypes of immunoglobulin such as IgG, A and E during subsequent encounters with the antigen (secondary response). IL-4 and IL-5 are the main cytokines that stimulate the growth and differentiation of B-lymphcoytes after exposure to antigen. -Specifically, IL-4 is responsible for B-cell growth and isotype switching. IL-4 stimulates secretion of IgE and predisposes to type I (immediate) hypersensitivity reactions. It also stimulates differentiation of TH0 (naive) T-helpcer cells into Th2 helpers, thus increasing the Th2 subpopulation and the stimulus for the humoral immune response. -IL-5 is responsible for B-cell differentiation; it stimulates IgA production and eosinophil activity and is important for host defense against parasitic infections. a. IL-1 is made by macrophages. It activates naive TH0 lymphocytes and promotes their differentiation into Th1 and Th2 subpopulation. IL-1 is also an endogenous pyrogen. b. IL-2 is the first interleukin produced by T-cells after contact with antigen. It is secreted mainly by Th1 cells and stimulates development of CD4 T-helper cells, CD8+ cytotoxic cells and B cells. c. IL-3 stimulates growth and differentiation of bone marrow stem cells and is produced by T-helper cells. e. IL-10 helps regulate the balance between Th1 and Th2 subpopulations of T-helper cells. It is produced by Th2 lymphocytes and inhibits synthesis of interferon-g leading to decrease in the Th1 subpopulation f. IL-12 is synthesized by macrophages and stimulates growth and development of the Th1 subpopulation of T-helper cells.

759 A 4 y.o old male is exposed to latex gloves during a minor surgical procedure and is subsequently found to produce anti-latex IgM antibodies. Several months later he develops a severe allergic reaction to latex and is found to have high level of serum anti-latex IgE antibodies. Which of the following cytokines is most likely responsible for this anti-latex antibody isotope change? a. IL-1 b. IL-2 c. IL-3 d. IL-4 e. IL-10 f. IL-12

contractile mechanism in skeletal muscle depends on calcium ions and proteins (myosin II, actin, tropomyosin, and troponin). Thick filaments in skeletal muscle are comprised of myosin molecules, with the heads of the myosin molecules forming cross links with actin during muscle contraction. The actin chains composed the thin filaments in the skeletal muscle. Tropomyosin sits in the groove between the two actin chains, covering the myosin binding sites on actin when the muscle is at rest. Troponin molecules are small globular proteins situated alongside the tropomyosin molecules which are comprised of three subunits: T, I and C. T binds the other troponin components to tropomyosin, troponin I binds to the troponin-tropomyosin complex to actin, and troponin C contains the bind sites for calcium. During excitation contraction coupling, Ca is released from the SR. When Ca binds to troponin C, tropomyosin shifts to expose the actin binding sites for myosin, allowing contraction to occur. a. Ach is a NT that initiates muscle contraction in response to motor neuron stimulation. Act release form the motor neuron opens post-synaptic ligand-gated ion channels, resulting in depolarization of the muscle cell Depolarization then causes release of Ca from the SR. b. Epinephrine is a catecholamine that is not directly involved in skeletal muscle contraction. However, epinephrine stimulates beta2-adrenergic receptors to increase skeletal muscle blood flow, glycogenolysis and lipolysis. c/d: calmodulin and MLCK are components of smooth muscle contractile mechanism.

7592 Physiologist are studying the bimolecular mechanisms underlying skeletal muscle contraction. They have been analyzing muscle fibers obtained from knockout mice to determine how different cellular substances influence muscular contraction. Striated muscle fibers obtaind from a specific mouse embryo fail to contract in response to a substance normally related from the sarcoplasmic reculum. A def of which of the following would best explain this finding? a. ACh b. Epinephrin c. calmodulin d. myosin light chain kinase e. troponin

Patient has hypertrophic cardiomyopathy, an AD disorder resulting form mutation sin cardiac sarcomere proteins. It is characterized by 1) ventricular septal hypertrophy 2) dynamic left ventricular outflow tract obstruction. Systolic anterior motion of the mitral valve toward the inter ventricular septum can cause eccentric mitral regurgitation and exacerbate LVOT obstruction. Patients can presents with exertion dyspnea, chest pain, fatigue, palpitations, dizziness, syncope and sudden cardiac death (d/t ventricular arrhythmias). Examination often reveals a harsh crescendo-decrescendo systolic murmur at the apex and left lower sternal border, which ranges in intensity with increase murmur intensity with low preload maneuvers and decrease murmur intensity with high preload and after load physiologic maneuver.

76 A 24 y.o man is being evaluated after an episode of syncope. He was jogging when he left lightheaded and passed out, but he didn't not sustained any head injury. patient had 2 similar episodes of lightheadedness while jogging over the last year, but this is the first time he passed out. He considered himself in good health and has not other medical problems. FH father died suddenly at 30. On PE, he has a harsh systolic murmur. Transthoracic echocardiography shows asymmetric inter ventricular septal hypertrophy. Patient's symptoms are most likely explained by left ventricular outflow obstruction created by which of the following structure? a. aortic valve cusp and inter ventricular septum a. aortic valve cusp and papillary muscle c. aortic valve cusps d. ascending aorta and inter ventricular septum e. mitral valve leaflet and inter ventricular septum

Tetanospasmin is a protein toxin produced by C. tetani that can travel by retrograde axonal transport into the CNS. The heavy chain of tetanospasmin binds to ganglioside receptors on neuronal membrane and the light chain inhibits release of glycine and GABA from inhibitory interneurons. Absence of these inhibitory NTs causes sustained muscle contraction or tetanus. Symptoms: -lock jaw -opisthotonos -dysphagia -facial muscle spasm (rises sardonic) Tetanus is prevented by the tetanus toxic (formaldehyde-inactivated tetanus toxin) vaccination. This vaccination elicits humoral immunity specific for the tetanus toxin. Tetanus immune globulin can administered during the acute treatment of tetanus and for tetanus prophylaxis after an injury if the town is grossly contaminated. a. CD 8 lymphocytes plays an important role in viral infections. They are capable of destroying tumor cells and participate in rejection reactions b. activated macrophages defend against intracellular organism and are principal cell type responsible for granuloma formation c. neutrophils is responsible for phagocytosis of foreign organism. these cells play a role in non-specific innate immunity and are not effective in preventing tetanus d. Immune defense against the majority medically important bacteria is provided by circulating antibodies that bind to microbial cells. Bound antibodies signal both macrophage phagocytosis and complement fixation, causing bacterial death. However, tetanus disease is the result of a protein toxin, not circulating C. tetani, thus, this defense mechanism is not effective against C. tetani.

760 A 7 y.o male injures his leg while playing in the yard. He has no significant PMH and all vaccinations are up to date. Several days after the injury, anaerobic cultures of a wound aspirate grow C. tetani. The patient never develops tetanus because of which of the following? a. sensitized CD 8 lymphocytes capable of killing the bacteria b. activated macrophages capable of killing the bacteria c. neutrophil infiltration of the wound with lysosomal enzyme release d. circulating antibodies that activate complement e. circulating antibodies the neutralize bacterial product

Th0 (naive CD4) secretes: 1. IFN-y and IL-12 to activate Th1 to secrete IFN-y, IL-2 and Lymphotoxin beta. -activation of macrophages and CD8+ T-cells (cytotoxic). -Mediates delayed type IIV hypersensitivity 2. IL-4 activates Th2 to secrete IL-4,5,10 and 13. -initiation of antibody response -regulation of Ig class switching -Activate B cells Th0 has T-cell receptor as well as CD3 and CD4. When antigen-MHCII complex is presented to naive Th by APC, both the TCR and CD4 proteins interact with antigen MHCII, while CD3 transmit the signal to the cell's interior. Antigen presented macrophage will produce IL-12 that stimulates differentiation of Th1-subset. IL-4 related by other types of APC facilitates the differentiation of Th2.

762 A 5 y.o boy has severe recurrent respiratory infections. A specific type of bacteria causes his infections. Further testing reveals that the patient's T-lymph are found to lack IL-12. Supplementation with which of the following substances would be more likely to improve this patient's condition? a. Ig b. erythropoietin c. IL-4 d. GM_CSF e. IFN-y f. early complement components

Phosphatidylcholine (aka lecithin( is a component of pulmonary surfactant and sphingomyelin is a common membrane phospholipid. A commonly used measure of fetal lung maturity is the amniotic fluid lecithin/sphingomyelin (L/S_ ratio. The amniotic fluid concentration of lecithin approximatelyy equals that of sphingomyelin until the middle of the 3rd trimester, at which point mature type II pneumocytes begin secreting surfactant. The lecithin concentration the increases sharply while the sphingomyelin level remains unchanged. By 35 weeks gestation, the L/S ratio averages 2:1 or higher, indicating lung maturity. Phosphatidylglycerol is a component of surfactant. It levels in amniotic fluid begins to increase after 36 weeks of gestation, in parallel with increased surfactant production by the maturing fetal lung. During their 3rd trimester, amniotic fluid sphingomyelin levels remain unchanged, as sphingomyelin is a common membrane phospholipid and only a minor component of pulmonary surfactant. Amniotic fluid total protein and albumin concentrations normally decrease by about 50% from early gestation to term.

800 Describe the amniotic fluid concentration of albumin, phosphatidylglycerol, phosphatydlcholine and sphingomyelin as you progress throughout gestation.

Diverge = no 3rd or 4th pharyngeal pouch derivatives = thyme aplasia = immunodeficiency from extremely low number of mature T lymph. -T-lymph are made in bone marrow but mature in thymus. Therefore patients are predisposed to recurrent infections by viral, fungal, protozoan and intracellular bacterial pathogens. -paracortex is the region of the lymph node where T lymph and dendritic cells chill. It lies internal to the cortex, between the follicles and medulla Dendritic would present antigens they collect form blood and lymph to the aggregated T-lymph in this region. The paracrotex becomes enlarged by the proliferation of T lymph during adaptive cellular immune response (eg, viral infections). In DiGeorge syndrome, this region is poorly developed d/t def of mature t lymph. a. Located in the outer cortex, the follicles are sites of B lymphocyte localization and proliferation. -Primary follicles: dense and dormant -Secondary follicles: pale germinal center containing proliferating B cell and folliculrlar dendritic cells. In agammaglobulinemia, germinal centers and primary lymphoid follicles do not form d/t an absence of B cells b/c. In the medulla you have cords and sinuses. The former contain B cells, plasma cells and macrophages, while the latter only have reticular cells and macrophages. e. the sub capsular sinus are the regions lying between the capsule and cortex of the lymph node. They are in direct communication with afferent lymphatic vessels and the cortical sinuses that line the trabecular. the cortical sinuses are continuous with medullary sinuses and the single efferent lymphatic channel existing the hilum.

8532 Patient has DiGeorge. Examination of his lymph nodes will most likely show poor development of which of the following structures? a. cortical follicles b. medullary sinus c. medullary cords d. paracortex region e. sub capsular sinuses.

Most inhaled anesthetics, barbiturates, and bentos achieve CNS depression by influencing GABA receptors and increasing the inhibitory action of GABA. Inhaled anesthetics have also been shown to affect potassium channels in the neuronal membranes and lock them in the state of hyperpolariation. They may also affect nicotinic glycine receptors. Apart from their desired action on CNS, inflation has affect on almost all organ systems of the body: 1. CVS: decrease in CO and increase in atrial and ventricular pressures. Hypotension associated with fluorinated anesthetics is a result of decrease in CO. 2. Respiratory: all inflation anesthetics, except NO, are respiratory depressants. They decrease tidal volume and minute ventilation and cause hypercapnia. Another undesirable effect is suppression of mucciliary clearance, which may predispose to postoperative atelectasis. Halothane and sevoflurane have bronchodilation properties and are preferred in patients with asthma. 3. in the brain, fluorinated anesthetics decrease vascular resistance and lead to an increase in cerebral blood flow. It is an undesirable effect because it causes increased intracranial pressure. 4. in kidneys, inhaled anesthetics decrease GFR, increase renal vascular resistance and decrease RPF 5. Fluorinated anesthetics decrease hepatic blood flow. a/b: renal function in general anesthesia is characterized by decrease if GFR and decrease in RPF c. fluorinated anesthetics are myocardial depressants. They decrease CO which leads to hypotension d. fluorinated anesthetic decrease hepatic blood flow.

854 A 36 y.o male has major surgical procedure under general anesthesia. He was given isoflurane to achieve a desirable depth of CNS depression. An increase in which of the following parameters is most likely to happen during the anesthesia in this patient? a. GFR b. effective RPF c. LV EF d. hepatic blood flow e. cerebral blood flow

The spinal cord is a round-to-oval structure containing a central "butterfly-like" area of gray matter surrounded by white matter. The gray matter is divided into the ventral (motor) horns and dorsal (sensory) horns. Features of the cord vary in spinal level and include the amount of white matter (in comparison to gray matter), the shape of the second (round vs. oval), the size of the ventral horns and the presence of lateral horns. Moving rostrally: -The amount of white matter steadily increases and the sections become more ovoid (particularly in the upper thoracic and cervical regions). Moving caudally -The ventral horns of the lower cervical and lumbosacral regions are more prominent as these areas innervate the muscles of the arms and legs, respectively. The amount of white matter appears larger in relation to the rather diminutive amount of gray matter. A feature that tis unique to thoracic and early lumbar levels (T1-L2) is the presence of of lateral horns (intermediolateral cell column), which are made of sympathetic preganglionic neurons. Gracile and cuneate fasciuculi are present above the T7 spinal level, whereas only the fragile fascicles is present below this level.

8635 How would you differentiate cross section of a spinal cord to see whether it is C1, C7, L5, S3 and T4?

Prolactinomas develop from lactotroph cells in the anterior pituitary (adenohypophysis), which is derived from surface ectoderm during embryonic development. Gastrulation = ectoderm, mesoderm and endoderm, which begins during week 3 of embryogenesis. -Initiated by primitive streak (a thickening of the epiblast cell layer that appears at the caudal end of the embryo and grows crinkly). Epiblast cells than undergo an epithelial-to-mesenchymal transition, causing them to lose their cell-cell adhesion properties, thus allow them to migrate downward through the primitive streak to form the mesoderm and endoderm layers. the remainder of the epiblast layer forms the ectoderm. -2 days after gastrulation begins, some of the epiblast cells migrate cranially through the primitive node through th epirmive node (the cephalic end of the primitive streak) to form a midline cellular cord known as the NOTOCHORD, which induces the overlying ectoderm to differentiate into neuroectoderm and form the neural plate. i. neurplate gives rise to neural tube and neural crest cells, while the remainder of the ectoderm becomes the surface ectoderm. e. the notochord becomes nucleus pulpous of the intervertebral disk in adults.

8702 Patient has a pituitary prolactinoma. Which of the following embryologic layers doesn't this lesion most likely originate from? a. endoderm b. mesoderm c. neural crest d. neural tube e. notochord f. surface ectoderm

Regulation of the hypothalamus-pituitary-thyroid axis: -The hypothalamus releases TRH triggering TSH to be released from pituitary. -TSH stimulates release thyroid T4 (thyroxine) and T3 (triiodothyronine), the latter being more active that are produced primarily via deiodination of T4 in peripheral tissues. Hypothyroidism may occur rd/t to 1. dysfunction involving the thyroid gland (primary hypothyroidism, most common) 2. Disorders of pituitary (secondary) 3. Hypothalamus (tertiary) The number one cause is d/t Hashimoto, which is what this person has. It is an autoimmune destruction of the thyroid gland. As this happens, there will be low levels of thyroid hormones, which causes an elevated level of TSH d/t lack of inhibition from the thyroid hormones. However, *T3 levels usually remain normal until relatively late-stages hypothyroidism.* TSH and T4 is normal. Total T3 increase -thyroid hormones circulate mostly in protein bound form primarily to thyroxine biding globulin, transthyretin and albumin. When TBG levels are elevated (eg, pregnancy or oral contraceptive use), total thyroid hormone levels are high but the free hormone levels are normal. Their TSH is also normal. Euthyroid. Elevated levels of TSH, T4 and T3 are consistent with hyperthyroidism d/t a TSH secreting pituitary adenoma. Secondary hypothyroidism will have decreased everything. Suppressed TSH will elevate thyroid hormone levels are characteristic of thyrotoxicosis (eg,Graves disease).

983 A 30 y.o woman has 3 week history of mild fatigue, cold intolerance, and constipation. She also has dry skin, weight gain, and myalgia. Mother had same thing and was diagnosed with thyroid disorder around the same age. PMH and SH are unremarkabale. She has been pregnant once and gave birth to a healthy infant 3 years ago. PE shows dry skin delayed ankle jerk reflexes and bradycardia. He thyroid gland is mild diffuse enlargement, with no tenderness or nodules. Labs of TSH, free T4 and total T3 would reveal what>

Alkaline phosphatase is in a bunch of places. Mostly bone and liver. To see whether the elevated ALP is from hepatic or bony origin, we must check y-glutamyl transpeptidase (not found in bone). a. unconjugated bilirubin level especially used in contrast to conjugated bilirubin can be use to distinguish hemolytic conditions from hepatic dysfunction or bile duct obstruction. b. lactate DH is nonspecific and can be use of some assistance in the valuation of tissue injury or death. c. NH3 levels can be use to determine the extent of liver failure or GI bleeding. d. PT is assessing coagulative ability and liver functionality, especially in acute setting.

99 A 40 y.o female has worsening fatigue. Lab shows an increase in alkaline phosphatase level of 180 U/L. Which of the following should we check next? a. unconjugated bilirubin level b. lactate DH c. serum NH3 d. PT e. y-glutamyl transpeptidase f. aspartate aminotransferase.

Normal vitamin D metabolism Dietary intake gives you D2 (ergocalciferol) and D3 (cholecalciferol). The UV from the sun converted 7-dehydrocholesterol in the skin into D3. D2/D3 gets converted by 25-hydroxlase in the liver into 25-hydroxyvitamin D. 25-OH vitamin D gets converted by 1-alpha-hydroxylase (unregulated by PTH and inhibited by calcium) into 1,25 OH Vitamin D. This active form of vitamin D does a couple of things: 1. Bone: increased mineralization 2. SI: increased calcium and phosphate absorption (major effect) 3. Kidneys: increase calcium and phosphate reabsorption 4. PTH glands: decrease PTH secretion

990 A 49 y.o man complains of aching bones. He has a 2 month history of insidious onset pain that is most pronounced in his back, pelvis and lower extremities. The pain is dull and increases after weight bearing activities. The patient emigrated from Africa 5 years ago and works overnight shifts as a cab driver. After a thorough evaluation the patient is advised tp spend about 15 minutes a day outdoors. Which step in vitamin D metabolism will be directly affected by this recommendation?

Dude has HBV -unprotected sex (can also be transmitted percutaneous (eg IV drug users, needlestick accident and blood transfusion) and vertical transmission -unvaccinated -liver function abnormalities -tender hepatomegaly c. anti-HBsAg IgG appears after successful HBV vaccination or HBsAg clearance and remains detectable for life (an indicator of non-infectivity and immunity). Because this patient is symptomatic he most likely doesn't have anti-HBsAg Ig. a/b: HAV in stool or serum can be seen in acute HAV following recent travel to an endemic area or consumption of tainted, insufficiently heated food (eg, posters). Transmission is via fecal-oral route. Not usually symptomatic for 2 months. It is not associated with chronic carrier state. e. HVC is most often asymptomatic. Transmission mostly through IV drug use, sexual transmission can happen but risk is very low (0.07% per year) and would be lower than that of HBV transmission. f. HDV transmitted mostly through inoculations and blood transfusions. Co-infection with HBV or superinfection of chronic HBV is necessary

A 23 y.o dude has a 2 month history of fatigue, malaise, and abdominal discomfort. PE tender hepatomegaly with elevated liver function tests. Never been vaccinated. Smokes 2 packs of cigs a day. Has had several episodes of unprotected sex with different female partners within the past year. What would be found in his lab work up? a. HAV in stool? b. serum anti-HAV IgM c. serum anti-HBsAg IgG d. serum HBsAg e. Hep C virus RNA viral load in blood f. serum anti-hepatitis D virus IgG


Kaugnay na mga set ng pag-aaral

Rounding to the nearest Thousand

View Set

Chemistry Exam - Chapters 10 and 11

View Set

Healthy Lifestyles Ch 8 Quiz Questions

View Set